Sie sind auf Seite 1von 44

Endocrinology

Question 1 Which one of the following is a recognised cause of hypokalaemia associated with hypertension A. Liddle's syndrome B. Bartter's syndrome C. Gitelman syndrome . Ciclosporin !. "enal tu#ular acidosis Liddle's syndrome$ hypokalaemia % hypertension Liddle's syndrome is an autosomal dominant disorder that mimics hyperaldosteronism& resulting in hypokalaemia associated with hypertension 'ypokalaemia and hypertension (or postgraduate e)ams it is useful to #e a#le to classify the causes of hypokalaemia in to those associated with hypertension& and those which are not 'ypokalaemia with hypertension Cushing's syndrome Conn's syndrome *primary hyperaldosteronism+ Liddle's syndrome 11,#eta hydro)ylase deficiencyCar#eno)olone& an anti,ulcer drug& and li.uorice e)cess can potentially cause hypokalaemia associated with hypertension 'ypokalaemia without hypertension diuretics G/ loss *e.g. diarrhoea& 0omiting+ renal tu#ular acidosis *type 1 and 1--+ Bartter's syndrome Gitelman syndrome -11,hydro)ylase deficiency& which accounts for 234 of congenital adrenal hyperplasia cases& is not associated with hypertension --type 5 renal tu#ular acidosis is associated with hyperkalaemia Question 1 A 16,year,old girl is in0estigated for primary amenorrhoea& despite ha0ing de0eloped secondary se)ual characteristics at 11 years of age. 7n e)amination she has well de0eloped #reasts with scanty pu#ic hair and small #ilateral groin swellings. What is the most likely diagnosis8 A. Congenital adrenal hyperplasia B. 9olycystic o0arian syndrome C. :urner's syndrome . :esticular feminsation syndrome !. ;ullerian duct agenesis :esticular feminisation syndrome :his <,linked recessi0e condition is due to end,organ resistance to testosterone causing genotypically male children *5=<>+ to ha0e a female phenotype (eatures 'primary amennorhoea' undescended testes causing groin swellings #reast de0elopment may occur as a result of con0ersion of testosterone to oestradiol iagnosis #uccal smear or chromosomal analysis to re0eal 5=<> genotype ;anagement counselling , raise child as female #ilateral orchidectomy *increased risk of testicular cancer due to undescended testes+ oestrogen therapy Question ? A 56,year,old female is admitted to the !mergency epartment with a#dominal pain associated with 0omiting. @he has a past medical history of hypothyroidism and takes thyro)ine. 7n e)amination she is pyre)ial at ?A.=BC. 9ulse is 113 #pm with a #lood pressure of 133C=5 mm'g. Blood results show the following$ Da% 1?1 mmolCl E% 5.2 mmolCl Frea G.1 mmolCl Creatinine 113 HmolCl Glucose ?.? mmolCl What treatment should #e gi0en first8 A. Ceftria)one % #enIylpenicillin B. Glucagon C. 9ropranolol . :riiodothyronine

!. 'ydrocortisone :his is a typical history of Addison's. 9atients may ha0e a history of other autoimmune conditions such as thyroid disorders. @teroids should #e gi0en as soon as possi#le Addison's disease Autoimmune destruction of the adrenal glands is the commonest cause of hypoadrenalism in the FE& accounting for G34 of cases (eatures lethargy& weakness& anore)ia& DCJ& weight loss hyperpigmentation& 0itiligo& loss of pu#ic hair in women crisis$ collapse& shock& pyre)ia 7ther causes of hypoadrenalism 9rimary causes tu#erculosis metastases *e.g. #ronchial carcinoma+ meningococcal septicaemia *Waterhouse,(riderichsen syndrome+ '/J antiphospholipid syndrome @econdary causes pituitary disorders *e.g. tumours& irradiation& infiltration+ !)ogenous glucocorticoid therapy Question 5 A 6? year man presents as his wife has noticed a change in his appearance. 'e has also noticed his hands seem larger. 7n e)amination #lood pressure is 1A3C25 and he is noted to ha0e #itemporal hemianopia. What is the most appropriate first, line treatment8 A. 7ctreotide B. !)ternal irradiation C. 9eg0isomant . :rans,sphenoidal surgery !. Bromocriptine :rans,sphenoidal surgery is the treatment of choice in acromegaly. :here is no significant e0idence #ase supporting the use of pre,operati0e octreotide Acromegaly$ management :rans,sphenoidal surgery is first,line treatment for acromegaly in the maKority of patients opamine agonists for e)ample #romocriptine the first effecti0e medical treatment for acromegaly& howe0er now superseded #y somatostatin analogues effecti0e only in a minority of patients @omatostatin analogue for e)ample octreotide effecti0e in 63,A34 of patients may #e used as an adKunct to surgery 9eg0isomant G' receptor antagonist , pre0ents dimeriIation of the G' receptor once daily sCc administration 0ery effecti0e , decreases /G(,1 le0els in 234 of patients to normal doesn't reduce tumour 0olume therefore surgery still needed if mass effect !)ternal irradiation is sometimes used for older patients or following failed surgicalCmedical treatment Question 6 A 6=,year,old man is re0iewed in the Cardiology outpatient clinic following a myocardial infarction one year pre0iously. uring his admission he was found to #e hypertensi0e and dia#etic. 'e complains that he has put on 6kg in weight in the past = months. Which of his medications may #e contri#uting to his weight gain8 A. ;etformin B. Losartan C. Clopidogrel . GlicaIide !. @im0astatin @ulfonylureas @ulfonylureas are oral hypoglycaemic drugs used in the management of type 1 dia#etes mellitus. :hey work #y increasing pancreatic insulin secretion and hence are only effecti0e if functional B,cells are present. Common ad0erse effects hypoglycaemic episodes *more common with long acting preparations such as chlorpropamide+

increased appetite and weight gain "arer ad0erse effects syndrome of inappropriate A ' secretion #one marrow suppression li0er damage *cholestatic+ photosensiti0ity peripheral neuropathy @ulfonylureas should #e a0oided in #reast feeding and pregnancy Question = !ach one of the following is a cause of cranial dia#etes insipidus& e)cept$ A. 9ituitary surgery B. Lithium C. 'istiocytosis < . Craniopharyngioma !. 9ost head,inKury Lithium causes a nephrogenic dia#etes insipidus ia#etes insipidus Causes of cranial / idiopathic post head inKury pituitary surgery craniopharyngiomas histiocytosis < / ;7A is the association of cranial ia#etes /nsipidus& ia#etes ;ellitus& 7ptic Atrophy and eafness *also known as Wolfram's syndrome+ Causes of nephrogenic / genetic *primary+ electrolytes$ hypercalcaemia& hypokalaemia drugs$ demeclocycline& lithium tu#ulo,interstitial disease$ o#struction& sickle,cell& pyelonephritis /n0estigation high plasma osmolarity& low urine osmolarity water depri0ation test Question A Which one of the following statements regarding maturity,onset dia#etes of the young *;7 >+ is true8 A. :here is usually a strong family history B. Body mass inde) is typically L ?3 C. oesnMt respond to glimepiride . Autosomal recessi0e inheritance !. (re.uent episodes of dia#etic ketoacidosis are typical ;7 > ;aturity,onset dia#etes of the young *;7 >+ is characterised #y the de0elopment of type 1 dia#etes mellitus in patients N 16 years old. /t is typically inherited as an autosomal dominant condition. 70er si) different genetic mutations ha0e so far #een identified as leading to ;7 >. Eetosis is not a feature at presentation ;7 > ? =34 of cases due to a defect in the 'D(,1 alpha gene ;7 > 1 134 of cases due to a defect in the glucokinase gene Question G What causes increased sweating in patients with acromegaly8 A. /ncreased sodium content in sweat B. ecreased L' and (@' secondary to hypopituitarism C. !pisodic hypoglycaemia . Low,grade chronic pyre)ia !. @weat gland hypertrophy

Acromegaly$ features /n acromegaly there is e)cess growth hormone secondary to a pituitary adenoma in o0er 264 of cases. A minority of cases are caused #y ectopic G'"' or G' production #y tumours e.g. pancreatic (eatures coarse& oily skin & large tongue& prognathism& interdental spaces spade,like hands& increase in shoe siIe features of pituitary tumour$ hypopituitarism& headaches& #itemporal hemianopia raised prolactin in 1C? of cases ,,L galactorrhoea =4 of patients ha0e ;!D,1 Complications hypertension dia#etes *L134+ cardiomyopathy colorectal cancer Question 2 A A,year,old #oy is #eing in0estigated for failure to thri0e and generalised weakness. 'is #lood pressure is normal. :he following #lood results are o#tained$ Da% 1?A mmolCl E% ?.3 mmolCl Frea 5.6 mmolCl Creatinine =6 HmolCl Bicar#onate?? mmolCl What is the most likely diagnosis8 A. Conn's syndrome B. Bartter's syndrome C. Cushing's syndrome . 11,hydro)ylase deficiency !. Liddle's syndrome Bartter's syndrome is associated with normotension Bartter's syndrome is the most likely diagnosis. Congenital adrenal hyperplasia due to 11,hydro)ylase deficiency is associated with precocious pu#erty rather than failure to thri0e in #oys. Both Conn's and Cushing's are associated with hypertension and are not common in this age group Bartter's syndrome Bartter's syndrome is an inherited cause *usually autosomal recessi0e+ of se0ere hypokalaemia due to defecti0e chloride a#sorption at the Da% E% 1Cl, cotransporter in the ascending loop of 'enle. /t should #e noted that is associated with normotension *unlike other endocrine causes of hypokalaemia such as Conn's& Cushing's and Liddle's syndrome which are associated with hypertension+ (eatures usually presents in childhood& e.g. failure to thri0e polyuria& polydipsia hypokalaemia normotension weakness Question 13 A 56,year,old man is in0estigated following referral to the endocrinology clinic with polydipsia. 9lasma glucose and calcium are normal. A water depri0ation test is performed with the following results$ @tarting plasma osm. 121 m7smolCl (inal urine osm. 151 m7smolCl Frine osm. post, AJ9 GG6 m7smolCl What is the most likely diagnosis8 A. 9sychogenic polydipsia B. Dephrogenic dia#etes insipidus C. 9rimary hyperparathyroidism . 9seudohypoparathyroidism !. Cranial dia#etes insipidus A dramatic impro0ement is seen in the a#ility of the kidneys to concentrate urine following the administration of AJ9. :his points towards a diagnosis of cranial dia#etes insipidus Water depri0ation test ;ethod pre0ent patient drinking water ask patient to empty #ladder hourly urine and plasma osmolalities @tarting plasma osm. (inal urine osm. Frine osm. post, AJ9 Dormal Dormal L =33 L =33 9sychogenic polydipsia Low L 533 L 533 Cranial / 'igh N ?33 L =33 Dephrogenic / 'igh N ?33 N ?33

Question 11 A ?6,year,old female who has recently #eing diagnosed with Gra0e's disease presents for re0iew ? months after starting a '#lock and replace' regime with car#imaIole and thyro)ine. @he is concerned a#out de0eloping thyroid eye disease. What is the #est way that her risk of de0eloping thyroid eye disease can #e reduced8 A. "educe alcohol intake B. A diet rich in omega,? fatty acids C. "egular e)ercise . @top smoking !. Lose weight @moking is the most important modifia#le risk factor for the de0elopment of thyroid eye disease :hyroid eye disease :hyroid eye disease affects #etween 16,634 of patients with Gra0e's disease. /t is thought to #e due to an autoimmune response against an autoantigen& possi#ly the :@' receptor& causing retro,or#ital inflammation. :he patient may #e eu,& hypo, or hyperthyroid at the time of presentation 9re0ention smoking is the most important modifia#le risk factor for the de0elopment of thyroid eye disease radioiodine treatment may increase the inflammatory symptoms seen in thyroid eye disease. /n a recent study of patients with Gra0e's disease around 164 de0eloped& or had worsening of& eye disease. 9rednisolone may help reduce the risk (eatures e)ophthalmos conKuncti0al oedema papilloedema ophthalmoplegia ;anagement steroids radiotherapy surgery Question 11 A 5=,year,old man with suspected dia#etes mellitus has an oral glucose tolerance test& following the standard W'7 protocol. :he following results are o#tained$ :ime *hours+ Blood glucose *mmolCl+ 3 6.A 1 A.= 'ow should these results #e interpreted8 A. Dormal B. /mpaired fasting glucose and impaired glucose tolerance C. ia#etes mellitus . /mpaired glucose tolerance !. /mpaired fasting glucose ia#etes mellitus$ diagnosis Based on the World 'ealth 7rganisation 133= guidelines /f symptomatic fasting glucose L A.3 mmolCl random glucose L 11.1 mmolCl *or after A6g oral glucose tolerance test+ /f asymptomatic a#o0e criteria apply #ut must #e repeated on two occasions /mpaired fasting glucose and impaired glucose tolerance A fasting glucose LO =.1 #ut N A.3 mmolCl implies impaired fasting glucose */(G+ /mpaired glucose tolerance */G:+ is defined as fasting plasma glucose N A.3 mmolCl and 7G:: 1,hour 0alue LO A.G mmolCl #ut N 11.1 mmolCl ia#etes FE suggests$ '9eople with /(G should then #e offered an oral glucose tolerance test to rule out a diagnosis of dia#etes. A result #elow 11.1 mmolCl #ut a#o0e A.G mmolCl indicates that the person doesnMt ha0e dia#etes #ut does ha0e /G:.' Question 1? A 5=,year,old woman is referred to endocrine with a tender neck swelling. Blood results are as follows$ :@' N3.1 mFCl :5 1GG nmolCl '# 15.1 gCdl 9lt ?AA - 132Cl WBC =.5 - 132Cl !@" :echnetium thyroid scan shows decreased uptake glo#ally. What is the most likely diagnosis8 A. @ick thyroid syndrome B. Acute #acterial thyroiditis C. 'ashimoto's thyroiditis . @u#acute thyroiditis

=6 mmChr

!. :o)ic multinodular goitre @u#acute thyroiditis is suggested #y the tender goitre& hyperthyroidism and raised !@". :he glo#ally reduced uptake on technetium thyroid scan is also typical @u#acute thyroiditis *also known as e Quer0ain's thyroiditis+ is thought to occur following 0iral infection and typically presents with hyperthyroidism (eatures hyperthyroidism painful goitre raised !@" glo#ally reduced uptake on iodine,1?1 scan ;anagement usually self,limiting , most patients do not re.uire treatment thyroid pain may respond to aspirin or other D@A/ s in more se0ere cases steroids are used& particularly if hypothyroidism de0elops Question 15 A 56,year,old man is referred to the acute medical unit. 'e had presented earlier in the day to the G9 complaining of ongoing fatigue and polydipsia. A B; taking in the surgery was 11.?. 7n e)amination he is an o#ese man *B;/ ?=+ with a pulse of G5 #pm and #lood pressure of 155CG5 mm'g. Blood tests re0eal the following$ Da% 153 mmolCl& E% ?.2 mmolCl& Bicar#onate 11 mmolCl& Frea 6.1 mmolCl& Creatinine 131 HmolCl& Glucose 11.1 mmolCl What is the most appropriate initial management8 A. GlicaIide B. 9ioglitaIone C. Weight loss . ;etformin !. Commence insulin therapy Weight reduction alone would #e insufficient in this patient with frank dia#etes ;etformin is a #iguanide used mainly in the treatment of type 1 dia#etes mellitus. /t has a num#er of actions which impro0es glucose tolerance *see #elow+. Fnlike sulphonylureas it does not cause hypoglycaemia and weight gain and is therefore first,line if the patient is o0erweight. ;etformin is also used in polycystic o0arian syndrome and non,alcoholic fatty li0er disease ;echanism of action increases insulin sensiti0ity decreases hepatic gluconeogenesis may also reduce gastrointestinal a#sorption of car#ohydrates Ad0erse effects gastrointestinal upsets are common *nausea& anore)ia& diarrhoea+& intolera#le in 134 reduced 0itamin B11 a#sorption , rarely a clinical pro#lem Contraindicationslactic acidosis-- with se0ere li0er disease or renal failure do not use during suspected episodes of tissue hypo)ia *e.g. recent ;/& sepsis+ alcohol a#use is a relati0e contraindication stop 1 days #efore general anaesthetic& restart when renal function normal stop prior to /J contrast e.g. angiography& restart when renal function normal -metformin is now sometimes used in pregnancy& for e)ample in women with polycystic o0arian syndrome --it is now increasingly recognised that lactic acidosis secondary to metformin is rare& although it remains important in the conte)t of postgraduate e)ams Question 16 A =G,year,old woman is found to ha0e the following #lood tests$ :@' 3.36 muCl (ree :5 12 pmolCl *range 2,16 pmolCl+ (ree :? /f left untreated& what are the most likely possi#le conse.uences8 A. @upra0entricular arrhythmias and osteoporosis B. @upra0entricular arrhythmias and hyperlipidaemia C. 'ypothyroidism and impaired glucose tolerance . ;yasthenia gra0is and hypothyroidism !. /mpaired glucose tolerance and hyperlipidaemia

A pmolCl *range ?,2 pmolCl+

@u#clinical hyperthyroidism is an entity which is gaining increasing recognition. /t is defined as$ normal serum free thyro)ine and triiodothyronine le0els with a thyroid stimulating hormone *:@'+ #elow normal range *usually N 3.1 muCl+ Causes multinodular goitre& particularly in elderly females e)cessi0e thyro)ine may gi0e a similar #iochemical picture

:he importance in recognising su#clinical hyperthyroidism lies in the potential effect on the cardio0ascular system *atrial fi#rillation+ and #one meta#olism *osteoporosis+. /t may also impact on .uality of life and increase the likelihood of dementia ;anagement :@' le0els often re0ert #ack to normal , therefore le0els must #e persistently low to warrant inter0ention a reasona#le treatment option is a therapeutic trial of low,dose antithyroid agents for appro)imately = months in an effort to induce a remission Question 1= A 16,year,old Asian woman who is 1= weeks pregnant has an oral glucose tolerance test *7G::+. :his was re.uested due to a com#ination of her ethnicity and a #ackground of o#esity. :he following results are o#tained$ :ime *hours+ Blood glucose *mmolCl+ 3 6.6 1 11.1 What is the most appropriate management8 A. @tart insulin B. Gi0e ad0ice a#out a dia#etic diet C. Gi0e ad0ice a#out a dia#etic diet % repeat 7G:: in 5 weeks . @tart metformin !. @tart insulin % aspirin 9regnancy$ dia#etes mellitus ia#etes mellitus may #e a pre,e)isting pro#lem or de0elop during pregnancy& gestational dia#etes. /t complicates around 1 in 53 pregnancies "isk factors for gestational dia#etes B;/ of L ?3 kgCmP1 pre0ious macrosomic #a#y weighing 5.6 kg or a#o0e. pre0ious gestational dia#etes first,degree relati0e with dia#etes family origin with a high pre0alence of dia#etes *@outh Asian& #lack Cari##ean and ;iddle !astern+ @creening for gestational dia#etes , if a women has had gestational dia#etes pre0iously an oral glucose tolerance test *7G::+ should #e performed at 1=,1G weeks and at 1G weeks if the first test is normal women with any of the other risk factors should #e offered an 7G:: at 15Q1G weeks D/C! issued guidelines on the management of dia#etes mellitus in pregnancy in 133G ;anagement weight loss for women with B;/ of L 1A kgCmP1 stop oral hypoglycaemic agents& apart from metformin-& and commence insulin folic acid 6 mgCday from pre,conception to 11 weeks gestation detailed anomaly scan at 1G,13 weeks including four,cham#er 0iew of the heart and outflow tracts tight glycaemic control reduces complication rates treat retinopathy as can worsen during pregnancy Women who de0elop gestational dia#etes should stop taking hypoglycaemic medication following deli0ery. A fasting glucose should #e checked at the = week postnatal check -there is increasing e0idence that metformin is safe during pregnancy Question 1A A 5=,year,old man presents as he is concerned a#out reduced li#ido and e)cessi0e thirst. 7n .uestioning he also complains of pains in #oth hands. Which one of the following in0estigations is most likely to re0eal the diagnosis8 A. (erritin B. :estosterone C. Cortisol . Blood glucose !. 9rolactin :he a#o0e patient has symptoms consistent with haemochromatosis. :he e)cessi0e thirst is secondary to untreated dia#etes mellitus 'aemochromatosis is an autosomal recessi0e disorder of iron a#sorption and meta#olism resulting in iron accumulation. /t is caused #y inheritance of mutations in the '(! gene on #oth copies of chromosome =-. /t is often asymptomatic in early disease and initial symptoms often non,specific e.g. lethargy and arthralgia 9resenting features '#ronIe' skin pigmentation dia#etes mellitus li0er$ stigmata of chronic li0er disease& hepatomegaly& cirrhosis& hepatocellular deposition+ cardiac failure *1nd to dilated cardiomyopathy+ hypogonadism *1nd to cirrhosis and pituitary dysfunction , hypogonadotrophic hypogonadism+

arthritis *especially of the hands+ Questions ha0e pre0iously #een asked regarding which features are re0ersi#le with treatment$ "e0ersi#le complications Cardiomyopathy @kin pigmentation /rre0ersi#le complications Li0er cirrhosis-ia#etes mellitus 'ypogonadotrophic hypogonadism Arthropathy -there are rare cases of families with classic features of genetic haemochromatosis #ut no mutation in the '(! gene --whilst ele0ated li0er function tests and hepatomegaly may #e re0ersi#le& cirrhosis is not Question 1G Which one of the following is not associated with primary hyperparathyroidism8 A. 'ypotension B. ;ultiple endocrine neoplasia type 1 C. ;ultiple endocrine neoplasia type 1a . epression !. 9ancreatitis 9rimary hyperparathyroidism is associated with hypertension 9rimary hyperparathyroidism /n postgraduate e)ams primary hyperparathyroidism is stereotypically seen in elderly females with an un.uencha#le thirst and an inappropriately normal or raised parathyroid hormone le0el. /t is most commonly due to a solitary adenoma Causes of primary hyperparathyroidism G34$ solitary adenoma 164$ hyperplasia 54$ multiple adenoma 14$ carcinoma (eatures , '#ones& stones& a#dominal groans and psychic moans' polydipsia& polyuria peptic ulcerationCconstipationCpancreatitis #one painCfracture renal stones depression hypertension impaired glucose tolerance Associations hypertension multiple endocrine neoplasia$ ;!D / and // /n0estigations raised calcium& low phosphate 9:' may #e raised or normal technetium,;/B/ su#traction scan :reatment total parathyroidectomy Question 12 A 65,year,old woman presents to the !mergency epartment with confusion and fe0er. @he has a past history of thyroto)icosis pre0iously treated with radioiodine therapy. 7n e)amination she has a pulse of 113Cmin regular& #lood pressure 163C23 mm'g& temperature of ?2.1BC and a respiratory rate of 1GCmin. !)amination of the cardiorespiratory system is unremarka#le and urine dipstick is clear. Blood results showed the following$ (ree :5 G5 pmolCl *normal range 13Q11 pmolCl+ (ree :? 12 pmolCl *1.6Q6.6 pmolCl+ :@' N 3.31 mFCl *3.6Q5.3 mFCl+ Which one of the following does not ha0e a role in the su#se.uent management8 A. LugolMs iodine B. 9ropranolol C. 9ropylthiouracil . Bicar#onate !. e)amethasone :hyroid storm is a rare #ut life,threatening complication of thyroto)icosis. /t is typically seen in patients with esta#lished thyroto)icosis and is rarely seen as the presenting feature. /atrogenic thyro)ine e)cess does not usually result in thyroid storm

Clinical features include$ fe0er L ?G.6BC tachycardia confusion and agitation nausea and 0omiting hypertension heart failure a#normal li0er function test ;anagement symptomatic treatment e.g. paracetamol treatment of underlying precipitating e0ent anti,thyroid drugs$ e.g. methimaIole or propylthiouracil LugolMs iodine de)amethasone , e.g. 5mg /J .ds , #locks the con0ersion of :5 to :? propranolol Question 13 A =6,year,old man with a history of dyspepsia is found to ha0e a gastric ;AL: lymphoma on #iopsy. What treatment should #e offered8 A. Gastrectomy B. Laser a#lation C. Done . C'79 chemotherapy !. '. pylori eradication Gastric ;AL: lymphoma 70er0iew associated with '. pylori infection in 264 of cases good prognosis if low grade then G34 respond to '. pylori eradication (eatures paraproteinaemia may #e present Question 11 A A1,year,old woman with a history of type 1 dia#etes mellitus presents with lethargy and polyuria. A diagnosis of hyperosmolar hyperglycaemic state is considered. Which one of the following findings would #e least consistent with this diagnosis8 A. p' of A.?G B. Eetones 1% in urine C. @erum osmolality of ?13 mosmolCkg . @erum #icar#onate of 12 mmolCl !. Glucose of 56 mmolCl A trace of ketones may #e found in hyperosmolar hyperglycaemic state. @erum osmolality is typically L ?13 mosmolCkg 'yperosmolar hyperglycaemic state :he American ia#etes Association criteria for the diagnosis of hyperosmolar hyperglycaemic state *''@+ is as follows$ glucose L ??.? mmolCl p' L A.?3 serum #icar#onate L 16 mmolCl serum osmolality L ?13 mosmolCkg traces of ketones may #e present in urines Question 11 A 56,year,old man presents with #itemporal hemianopia and spade,like hands. What is the definite test to confirm the diagnosis8 A. !arly morning growth hormone B. /nsulin tolerance test C. 7ral glucose tolerance with growth hormone measurements . "andom insulin,like growth factor 1 */G(,R+ !. @hort AC:' test :he diagnostic test for acromegaly is an oral glucose tolerance with growth hormone measurements Acromegaly$ in0estigations Growth hormone *G'+ le0els 0ary during the day and are therefore not diagnostic.

:he definiti0e test is the oral glucose tolerance *7G::+ with serial G' measurements. @erum /G(,1 may also #e used as a screening test and is sometimes used to monitor disease 7ral glucose tolerance test in normal patients G' is suppressed to N 1 muCL with hyperglycaemia in acromegaly there is no suppression of G' may also demonstrate impaired glucose tolerance which is associated with acromegaly A pituitary ;"/ may demonstrate a pituitary tumour Question 1? What is the mechanism of action of rosiglitaIone8 A. 99A",gamma receptor antagonist B. 99A",alpha receptor antagonist C. 99A",alpha receptor agonist . 99A",gamma receptor agonist !. /ncreases endogenous insulin secretion GlitaIones are agonists of 99A",gamma receptors :hiaIolidinediones are a new class of agents used in the treatment of type 1 dia#etes mellitus. :hey are agonists to the 99A",gamma receptor and reduce peripheral insulin resistance :he 99A",gamma receptor is an intracellular nuclear receptor. /ts natural ligands are free fatty acids and it is thought to control adipocyte differentiation and function Ad0erse effects weight gain li0er impairment$ monitor L(:s fluid retention , therefore contraindicated in heart failure recent studies ha0e indicated an increased risk of fractures rosiglitaIone is not recommended for use in patients with ischaemic heart disease or peripheral arterial disease. :he risk of complications may #e increased if rosiglitIone is com#ined with insulin Question 15 A ?=,year,old female with a B;/ of ?5 kgCmP1 is re0iewed after managing to lose ? kg in the past month. @he asks a#out the possi#ility of starting a drug to help her lose weight. What is the primary mode of action of orilistat8 A. Leptin antagonist B. 9ancreatic lipase inhi#itor C. Blocks intestinal a#sorption of lipids . ';G,CoA reductase inhi#itor !. Centrally,acting appetite suppressant :he primary mode of action of orilistat is to inhi#it pancreatic lipases& which in turn will decrease the a#sorption of lipids from the intestine 7#esity$ therapeutic options :he management of o#esity consists of step,wise approach$ conser0ati0e$ diet& e)ercise medical surgical Orlistat is a pancreatic lipase inhi#itor used in the management of o#esity. Ad0erse effects include faecal urgencyCincontinence and flatulence. D/C! ha0e defined criteria for use of orlistat. /t should only #e prescri#ed as part of an o0erall plan for managing o#esity in adults who ha0e$ B;/ of 1G kgCmP1 or more with associated risk factors& or B;/ of ?3 kgCmP1 or more continued weight loss e.g. 64 at ? months normally used for N 1 year Sibutramine centrally acting appetite suppressant *inhi#itor of serotonin and noradrenaline uptake at hypothalamic sites that regular food intake+ ad0erse effects include$ hypertension *monitor #lood pressure and pulse during treatment+& constipation& headache& dry mouth& insomnia and anore)ia contraindicated if psychiatric illness& hypertension& /' & stroke& arrhythmias Rimonabant& a specific CB1 canna#inoid receptor antagonist& was withdrawn in 7cto#er 133G when the !uropean ;edicines Agency warned of serious psychiatric pro#lems including suicide Question 16 A 66,year,old woman is in0estigated following an osteoporotic hip fracture. :he following results are o#tained$ :@' N 3.36 muCl (ree :5 12 pmolCl Which one of the following autoanti#odies is most likely to #e present8

A. Anti,:@' receptor stimulating autoanti#odies B. Anti,nuclear anti#odies C. Anti,thyroglo#ulin autoanti#odies . Anti,microsomal anti#odies !. Anti,thyroid pero)idase autoanti#odies Anti,:@' receptor stimulating autoanti#odies *often referred to as :hyroid @timulating /mmunoglo#ulins+ are almost diagnostic of Gra0e's disease& the most common cause of thyroto)icosis in the FE Gra0e's disease$ features (eatures seen in Gra0e's #ut not in other causes of thyroto)icosis eye signs$ e)ophthalmos& ophthalmoplegia preti#ial my)oedema thyroid acropachy Autoanti#odies anti,:@' receptor stimulating anti#odies *234+ anti,thyroid pero)idase anti#odies *634+ Question 1= A 1A,year,old female de0elops eye pain and reduced 0isual acuity following the initiation of treatment for her recently diagnosed Gra0e's disease. Which one of the following treatments is likely to ha0e #een started8 A. "adioiodine treatment B. :hyroidectomy C. 9ropylthiouracil . Car#imaIole and thyro)ine !. Car#imaIole "adioiodine treatment may lead to the de0elopment C worsening of thyroid eye disease in up to 164 of patients with Gra0e's disease :hyroid eye disease affects #etween 16,634 of patients with Gra0e's disease. /t is thought to #e due to an autoimmune response against an autoantigen& possi#ly the :@' receptor& causing retro,or#ital inflammation. :he patient may #e eu,& hypo, or hyperthyroid at the time of presentation 9re0ention smoking is the most important modifia#le risk factor for the de0elopment of thyroid eye disease radioiodine treatment may increase the inflammatory symptoms seen in thyroid eye disease. /n a recent study of patients with Gra0e's disease around 164 de0eloped& or had worsening of& eye disease. 9rednisolone may help reduce the risk (eatures e)ophthalmos conKuncti0al oedema papilloedema ophthalmoplegia ;anagement steroids radiotherapy surgery Question 1A A 56,year,old man is re0iewed in the dia#etes clinic. :he following results are o#tained$ Frinalysis DA '#A1c G.=4 GlicaIide is added to the metformin he already takes. What is the minimum time period after which the '#A1c should #e repeated 8 A. = months B. 1 month C. 1 weeks . ? months !. 5 months '#A1C , recheck after 1,? months A more accurate answer would pro#a#ly #e 1 months #ut this is not gi0en as an option. @ee the e)planation #elow Glycosylated haemoglo#in *'#A1c+ is the most widely used measure of long,term glycaemic control in dia#etes mellitus. '#A1c is produced #y the glycosylation of haemoglo#in at a rate proportional to the glucose concentration. :he le0el of '#A1c therefore is dependant on red #lood cell lifespan a0erage #lood glucose concentration

'#A1c is generally thought to reflect the #lood glucose o0er the pre0ious '1,? months' although there is some e0idence it is weighed more strongly to glucose le0els of the past 1,5 weeks :he relationship #etween '#A1c and a0erage #lood glucose is comple) #ut has #een studied #y the ia#etes Control and Complications :rial 'BA1c *4+A0erage plasma glucose*mmolCl+ 6 6.6 = A.6 A 2.6 G 11.6 2 1?.6 13 16.6 11 1A.6 11 12.6 (rom the a#o0e we can see that a0erage plasma glucose O *1 - '#A1c+ , 5.6 Question 1G A 1G,year,old woman with polycystic o0arian syndrome consults you as she is ha0ing pro#lems #ecoming pregnant. @he has a past history of oligomenorrhea and has pre0iously recently stopped taking a com#ined oral contracepti0e pill. espite stopping the pill = months ago she is still not ha0ing regular periods. 'er #ody mass inde) is 1G kgCmP1. Apart from ad0ising her to lose weight& which one of the following inter0entions should #e used first,line to increase her chances of concei0ing8 A. ;etformin B. Bromocriptine C. Laparoscopic o0arian cautery . Clomifene !. 7rlistat /nfertility in 9C7@ , clomifene #efore metformin Whilst metformin has a role in the management of infertility it should #e used second,line to anti,oestrogens such as clomifene 9olycystic o0arian syndrome *9C7@+ is a comple) condition of o0arian dysfunction thought to affect #etween 6,134 of women of reproducti0e age. ;anagement is complicated and pro#lem #ased General weight reduction if appropriate if a women re.uires contraception then a com#ined oral contracepti0e *C7C+ pill may help regulate her cycle and induce a monthly #leed *see #elow+ 'irsuitism and acne a C7C pill may #e used help manage hirsuitism. 9ossi#le options include a third generation C7C which has fewer androgenic effects or co,cyprindiol which has an anti,androgen action. Both of these types of C7C may carry an increased risk of 0enous throm#oem#olism if doesn't respond to C7C then topical eflornithine may #e tried spironolactone& flutamide and finasteride may #e used under specialist super0ision /nfertility weight reduction if appropriate the management of infertility in patients with 9C7@ should #e super0ised #y a specialist D/C! recommend anti,oestrogens- such as clomifene and tamo)ifen as first,line treatment. 9otential risk of multiple pregnancies metformin is usually used second,line *either com#ined with clomifene or alone+& particularly in patients who are o#ese gonadotrophins -work #y occupying hypothalamic oestrogen receptors without acti0ating them. :his interferes with the #inding of oestradiol and thus pre0ents negati0e feed#ack inhi#ition of (@' secretion Question 12 /n patients with suspected insulinoma& which one of the following is considered the #est in0estigation8 A. 7ral glucose tolerance test B. /nsulin tolerance test C. !arly morning C,peptide le0els . Glucagon stimulation test !. @uper0ised fasting /nsulinoma is diagnosed with super0ised prolonged fasting C: of the pancreas is also useful in demonstrating a lesion

/nsulinoma Basics tumour of /slets of Langerhans most common pancreatic endocrine tumour 134 malignant& 134 multiple of patients with multiple tumours& 634 ha0e ;!D,1 (eatures of hypoglycaemia either early in morning or Kust #efore meal high insulin& raised proinsulin$insulin ratio high C,peptide iagnosis super0ised& prolonged fasting *up to A1 hours+ C: pancreas Question ?3 Which of the following secondary causes of hyperlipidaemia result in predominantly hypercholesterolaemia8 A. ia#etes mellitus B. BendrofluaIide C. Dephrotic syndrome . Alcohol !. 7#esity 'ypercholesterolaemia rather than hypertriglyceridaemia$ nephrotic syndrome& cholestasis& hypothyroidism 'yperlipidaemia$ secondary causes Causes of predominantly hypertriglyceridaemia dia#etes mellitus *types 1 and 1+ o#esity alcohol chronic renal failure drugs$ thiaIides& non,selecti0e #eta,#lockers& unopposed oestrogen li0er disease Causes of predominantly hypercholesterolaemia nephrotic syndrome cholestasis hypothyroidism Question ?1 A 56,year,old female is re0iewed in the medical clinic with a two month history of lethargy. Blood tests re0eal the following$ Da% 112 mmolCl E% 6.1 mmolCl Frea 6.? mmolCl Creatinine 22 HmolCl :otal :5 == mmolCl Which one of the following in0estigations is most likely to re0eal the diagnosis8 A. @erum glucose B. :@' C. (ree :5 . 70ernight de)amethasone suppression test !. @hort synacthen test :he short synacthen test is the #est test to diagnose Addison's disease 'yponatraemia and a high potassium in a patient with lethargy is highly suggesti0e of Addison's disease. :he thyro)ine le0el is slightly low and she may indeed ha0e co,e)isting hypothyroidism #ut this would not e)plain the high potassium ;any la#s ha0e an upper reference range for potassium of 6.6 mmolCl& #ut in the conte)t of the other results hypoadrenalism should #e suspected Addison's disease$ in0estigations /n a patient with suspected Addison's disease the definite in0estigation is a short AC:' test. 9lasma cortisol is measured #efore and ?3 minutes after gi0ing @ynacthen 163ug /;. Adrenal autoanti#odies such as anti,11,hydro)ylase may also #e demonstrated Associated electrolyte a#normalities hyperkalaemia hyponatraemia hypoglycaemia meta#olic acidosis Question ?1 A 12 year female presents a#dominal striae& hirsuitism and myopathy. A diagnosis of Cushing's syndrome is suspected. Which one of the following is the most appropriate test to confirm the diagnosis8 A. 'igh,dose de)amethasone suppression test B. 9lasma AC:' C. @hort AC:' test

. 15 hour urinary free cortisol !. 9etrosal sinus sampling of AC:' :he o0ernight de)amethasone suppression test is the #est test to diagnosis Cushing's syndrome :here is some de#ate as to whether a 15 hour urinary free cortisol or an o0ernight de)amethasone suppression test should #e used to screen patients for Cushing's. :he o0ernight de)amethasone suppression test has howe0er #een shown to #e more sensiti0e and is now much more commonly used in clinical practice. As this is not offered then 15 hour urinary free cortisol is the ne)t #est answer :he high,dose de)amethasone suppression test is used to help differentiate the cause of Cushing's syndrome Cushing's syndrome$ in0estigations /n0estigations are di0ided into confirming Cushing's syndrome and then localising the lesion. A hypokalaemic meta#olic alkalosis may #e seen& along with impaired glucose tolerance. !ctopic AC:' secretion *e.g. secondary to small cell lung cancer+ is characteristically associated with 0ery low potassium le0els. An insulin stress test is used to differentiate #etween true Cushing's and pseudo,Cushing's :ests to confirm Cushing's syndrome :he two most commonly used tests are$ o0ernight de)amethasone suppression test *most sensiti0e+ 15 hr urinary free cortisol Localisation tests :he first,line localisation is 2am and midnight plasma AC:' *and cortisol+ le0els. /f AC:' is suppressed then a non, AC:' dependent cause is likely such as an adrenal adenoma 'igh,dose de)amethasone suppression test if pituitary source then cortisol suppressed if ectopicCadrenal then no change in cortisol C"' stimulation if pituitary source then cortisol rises if ectopicCadrenal then no change in cortisol 9etrosal sinus sampling of AC:' may #e needed to differentiate #etween pituitary and ectopic AC:' secretion Question ?? An 1G,year,old girl is admitted to the !mergency epartment with an episode of sweating and diIIiness. @he is #rought in #y her father who has type 1 dia#etes mellitus as he is worried she may #e dia#etic. 'e descri#es a num#er of similar episodes for the past two weeks. 'er B; on admission is 1.2 mmolCl so the following #loods are taken$ 9lasma glucose 1.G mmolCl& /nsulin 16 mgCml *=,13 mgCml+&9roinsulin 114*11,154+&C,peptide3.16 nmolCl *3.1,3.5 nmolCl+ What is the most likely diagnosis8 A. ia#etes mellitus B. /nsulinoma C. Desidio#lastosis . /nsulin a#use !. @ulfonylurea a#use :he raised insulin with low c,peptide le0el points to a diagnosis of insulin a#use 'ypoglycaemia Causes insulinoma , increased ratio of proinsulin to insulin self,administration of insulinCsulphonylureas li0er failure Addison's disease alcohol 7ther possi#le causes in children nesidio#lastosis , #eta cell hyperplasia Question ?5 Which one of the following statements regarding polycystic o0arian syndrome *9C7@+ is incorrect8 A. A slightly ele0ated prolactin is consistent with a diagnosis of 9C7@ B. LuteiniIing hormone le0els are usually raised C. 'yperinsulinaemia is seen . Acanthosis nigricans may #e seen !. Affects #etween ?,64 of women of reproducti0e age 9olycystic o0arian syndrome is e)tremely common& affecting #etween 6,134 of women of reproducti0e age 9olycystic o0ary syndrome *9C7@+ is a comple) condition of o0arian dysfunction thought to affect #etween 6,134 of women of reproducti0e age. :he aetiology of 9C7@ is not fully understood. Both hyperinsulinaemia and high le0els of luteiniIing hormone are seen in 9C7@ and there appears to #e some o0erlap with the meta#olic syndrome (eatures su#fertility and infertility

menstrual distur#ances$ oligomenorrhea and amenorrhoea hirsuitism& acne *due to hyperandrogenism+ o#esity acanthosis nigricans *due to insulin resistance+ /n0estigations pel0ic ultrasound (@'& L'& prolactin& :@'& and testosterone are useful in0estigations$ raised L'$(@' ratio is a 'classical' feature #ut is no longer thought to #e useful is diagnosis. 9rolactin may #e normal or mildly ele0ated. :estosterone may #e normal or mildly ele0ated , howe0er& if markedly raised consider other causes check for impaired glucose tolerance Question ?6 Which one of the following may #e associated with galactorrhoea8 A. 9rimary hypothyroidism B. Addison's disease C. Cushing's syndrome . Gra0e's disease !. Bromocriptine 9rolactin and galactorrhoea 9rolactin is secreted #y the anterior pituitary gland with release #eing controlled #y a wide 0ariety of physiological factors. opamine acts as the primary prolactin releasing inhi#itory factor and hence dopamine agonists such as #romocriptine may #e used to control galactorrhoea. /t is important to differentiate the causes of galactorrhoea *due to the actions of prolactin on #reast tissue+ from those of gynaecomastia (eatures of e)cess prolactin men$ impotence& loss of li#ido& galactorrhoea women$ amenorrhoea& galactorrhoea Causes of raised prolactin prolactinoma pregnancy oestrogens physiological$ stress& e)ercise& sleep acromegaly$ 1C? of patients polycystic o0arian syndrome primary hypothyroidism *due to thyrotrophin releasing hormone *:"'+ stimulating prolactin release+ rug causes of raised prolactin metoclopramide& domperidone phenothiaIines haloperidol 0ery rare$ @@"/s& opioids Question ?= Which one of the following skin disorders is least associated with hypothyroidism8 A. <anthomata B. 9ruritus C. 9reti#ial my)oedema . !cIema !. ry& coarse hair (or the purposes of postgraduate e)ams preti#ial my)oedema is associated with thyroto)icosis. :here are howe0er case reports of it #een found in hypothyroid patients& especially the diffuse non,pitting 0ariety @kin disorders associated with thyroid disease @kin manifestations of hypothyroidism dry *anhydrosis+& cold& yellowish skin non,pitting oedema *e.g. hands& face+ dry& coarse scalp hair& loss of lateral aspect of eye#rows ecIema )anthomata @kin manifestations of hyperthyroidism preti#ial my)oedema$ erythematous& oedematous lesions a#o0e the lateral malleoli thyroid acropachy$ clu##ing scalp hair thinning increased sweating 9ruritus can occur in #oth hyper, and hypothyroidism Question ?A

A ??,year,old female is referred to endocrinology with thyroto)icosis. (ollowing a discussion of management options she elects to ha0e radioiodine therapy. Which one of the following is the most likely ad0erse effect8 A. 'ypothyroidism B. :hyroid malignancy C. Agranulocytosis . 7esophagitis !. 9recipitation of thyroid eye disease /t is well documented that radioiodine therapy can precipitate thyroid eye disease #ut a maKority of patients will e0entually re.uire thyro)ine replacement Gra0e's disease$ management espite many trials there is no clear guidance on the optimal management of Gra0e's disease. :reatment options include titration of anti,thyroid drugs *A: s& for e)ample car#imaIole+& #lock,and,replace regimes& radioiodine treatment and surgery. 9ropranolol is often gi0en initially to #lock adrenergic effects A: titration car#imaIole is started at 53mg and reduced gradually to maintain euthyroidism typically continued for 11,1G months patients following an A: titration regime ha0e #een shown to suffer fewer side,effects than those on a #lock,and,replace regime Block,and,replace car#imaIole is started at 53mg thyro)ine is added when the patient is euthyroid treatment typically lasts for =,2 months :he maKor complication of car#imaIole therapy is agranulocytosis "adioiodine treatment contraindications include pregnancy *should #e a0oided for 5,= months following treatment+ and age N 1= years. :hyroid eye disease is a relati0e contraindication& as it may worsen the condition the proportion of patients who #ecome hypothyroid depends on the dose gi0en& #ut as a rule the maKority of patient will re.uire thyro)ine supplementation after 6 years Question ?G Liddle's syndrome is associated with each one of the following& e)cept$ A. Alkalosis B. "esponse to treatment with amiloride C. 'ypertension . Autosomal recessi0e inheritance !. 'ypokalaemia Liddle's syndrome is a rare autosomal dominant condition that causes hypertension and hypokalaemic alkalosis. /t is thought to #e caused #y disordered sodium channels in the distal tu#ules leading to increased rea#sorption of sodium Treatment is with either amiloride or triamterene Question ?2 A 15,year,old female with a history of type 1 dia#etes mellitus presents to the !mergency epartment with 0omiting and a#dominal pain. (inger,prick testing estimates the #lood sugar to #e 16 mmolCl. Arterial #lood gases record a p' of A.11. 7n e)amination the patient is dehydrated and weighs G5 kg. An intra0enous line is sited and #loods are sent. 7ne litre of 3.24 saline is infused and an intra0enous insulin pump is set,up. What rate should insulin #e initially gi0en8 A. 13 unit C hour B. 1 unit C hour C. 1 unit C hour . G unit C hour !. = unit C hour ia#etic ketoacidosis :he most common precipitating factors of dia#etic ketoacidosis * EA+ are infection& missed insulin doses and myocardial infarction American ia#etes Association diagnostic criteria are as follows$ #lood glucose L1?.G mmolCl p' N A.?3 serum #icar#onate N1G mmolCl anion gap L 13 ketonaemia ;anagement fluid replacement$ most patients with EA are deplete around 6,G litres. /sotonic saline is used initially

insulin$ an intra0enous infusion should #e started at =uChour. 7nce #lood glucose is N 16 mmolCl an infusion of 64 de)trose should #e started correction of hypokalaemia Complications of EA and its treatment gastric stasis cere#ral oedema throm#oem#olism acute respiratory distress syndrome acute renal failure Question 53 A 1A,year,old man with multiple pigmented freckles on his lips and face is in0estigated for iron,deficiency anaemia. A diagnosis of 9eutI,Seghers syndrome is suspected. What is the mode of inheritance8 A. Autosomal recessi0e B. ;itochondrial inheritance C. <,linked dominant . Autosomal dominant !. <,linked recessi0e 9eutI,Seghers syndrome is an autosomal dominant condition characterised #y numerous hamartomatous polyps in the gastrointestinal tract. /t is also associated with pigmented freckles on the lips& face& palms and soles. Around 634 of patients will ha0e died from a gastrointestinal tract cancer #y the age of =3 years. Genetics autosomal dominant responsi#le gene encodes serine threonine kinase LEB1 or @:E11 (eatures hamartomatous polyps in G/ tract *mainly small #owel+ pigmented lesions on lips& oral mucosa& face& palms and soles intestinal o#struction e.g. intussusception gastrointestinal #leeding ;anagement conser0ati0e unless complications de0elop Question 51 A =5,year,old patient is prescri#ed peg0isomant for the treatment of acromegaly. What is the mechanism of action of peg0isomant8 A. /G(,1 receptor antagonist B. Growth hormone receptor antagonist C. /G(,1 receptor agonist . Growth hormone receptor agonist !. Long,acting somatostatin analogue Acromegaly$ management :rans,sphenoidal surgery is first,line treatment for acromegaly in the maKority of patients opamine agonists for e)ample #romocriptine the first effecti0e medical treatment for acromegaly& howe0er now superseded #y somatostatin analogues effecti0e only in a minority of patients @omatostatin analogue for e)ample octreotide effecti0e in 63,A34 of patients may #e used as an adKunct to surgery 9eg0isomant G' receptor antagonist , pre0ents dimeriIation of the G' receptor once daily sCc administration 0ery effecti0e , decreases /G(,1 le0els in 234 of patients to normal doesn't reduce tumour 0olume therefore surgery still needed if mass effect !)ternal irradiation is sometimes used for older patients or following failed surgicalCmedical treatment Question 51 Which one of the following features is not seen in carcinoid syndrome8 A. (lushing B. iarrhoea C. Bronchospasm . 'ypertension

!. 9ellagra (lushing& diarrhoea& #ronchospasm& tricuspid stenosis& pellagra ,,L carcinoid with li0er mets , diagnosis$ urinary 6,'/AA 'ypo, not hypertension is seen in carcinoid syndrome secondary to serotonin release Carcinoid syndrome usually occurs when metastases are present in the li0er and release serotonin into the systemic circulation may also occur with lung carcinoid as mediators are not 'cleared' #y the li0er (eatures flushing *often earliest symptom+ diarrhoea #ronchospasm hypotension right heart 0al0ular stenosis *left heart can #e affected in #ronchial carcinoid+ other molecules such as AC:' and G'"' may also #e secreted resulting in& for e)ample& Cushing's syndrome pellagra can rarely de0elop as dietary tryptophan is di0erted to serotonin #y the tumour /n0estigation urinary 6,'/AA plasma chromogranin A y ;anagement somatostatin analogues e.g. octreotide diarrhoea$ cyproheptadine may help Question 5? A 11,year,old female presents with recurrent painful oral ulceration. !)amination re0eals signs of oral Candidal infection. Which one of the following would most suggest type 1 polyglandular syndrome8 A. 'ypocalcaemia B. "heumatoid arthritis C. :ype // dia#etes mellitus . Coeliac disease !. 'ypercalcaemia 9rimary hypoparathyroidism is usually the first endocrine manifestation of type 1 autoimmune polyendocrinopathy syndrome. :he contrast to multiple endocrine neoplasia *;!D+& where hyperparathyroidism is a common finding& should #e noted :he .uestion gi0es a slightly atypical history as this is the upper end of the age range in which patients would #e e)pected to present Autoimmune polyendocrinopathy syndrome Addison's disease *autoimmune hypoadrenalism+ is associated with other endocrine deficiencies in appro)imately 134 of patients. :here are two distinct types of autoimmune polyendocrinopathy syndrome *A9@+& with type 1 *sometimes referred to as @chmidt's syndrome+ #eing much more common. A9@ type 1 has a polygenic inheritance and is linked to 'LA "?C "5. 9atients ha0e Addison's disease plus either$ type 1 dia#etes mellitus autoimmune thyroid disease A9@ type 1 is occasionally referred to as ;ultiple !ndocrine eficiency Autoimmune Candidiasis *;! AC+. /t is a 0ery rare autosomal recessi0e disorder caused #y mutation of A/"!1 gene on chromosome 11 (eatures of A9@ type 1 *1 out of ? needed+ chronic mucocutaneous candidiasis *typically first feature as young child+ Addison's disease primary hypoparathyroidism -Jitiligo can occur in #oth types Question 55 A 6=,year,old lady with a B;/ of 1A is re0iewed in the dia#etic clinic due to poor glycaemic control. @he is currently #eing treated with glicaIide 1=3mg #d. 'er latest #loods show$ Da% 1?2 mmolCl E% 5.1 mmolCl Frea G.5 mmolCl Creatinine 1=3 HmolCl AL: 16 iuCl yG: ?? iuCl '#A1c 2.54 Which one of the following medications should #e added ne)t8 A. Guar gum B. "osiglitaIone C. ;etformin . Acar#ose !. "epaglinide Gi0en that she is o0erweight metformin would #e a natural choice to add. :he creatinine howe0er is ele0ated so rosiglitaIone is the ne)t #est option 7ne possi#le draw#ack of using rosiglitaIone is that it may lead to weight gain& especially gi0en her B;/ is already 1A

:hiaIolidinediones are a new class of agents used in the treatment of type 1 dia#etes mellitus. :hey are agonists to the 99A",gamma receptor and reduce peripheral insulin resistance :he 99A",gamma receptor is an intracellular nuclear receptor. /ts natural ligands are free fatty acids and it is thought to control adipocyte differentiation and function Ad0erse effects weight gain li0er impairment$ monitor L(:s fluid retention , therefore contraindicated in heart failure recent studies ha0e indicated an increased risk of fractures rosiglitaIone is not recommended for use in patients with ischaemic heart disease or peripheral arterial disease. :he risk of complications may #e increased if rosiglitIone is com#ined with insulin Question 56 :he first,line treatment in remnant hyperlipidaemia is$ A.AFrsodeo)ycholic acidia B.AJitamin Aia C.A@tatinsia .A(ish oilia !.A(i#ratesia "emnant hyperlipidaemia 70er0iew rare cause of mi)ed hyperlipidaemia *raised cholesterol and triglyceride le0els+ also known as (redrickson type /// hyperlipidaemia& #road,#eta disease and dys#etalipoproteinaemia associated with apo,e1 homoIygosity high incidence of ischaemic heart disease and peripheral 0ascular disease thought to #e caused #y impaired remo0al of / L from the circulation #y the li0er (eatures yellow palmar creases palmer )anthomas tu#erous )anthomas ;anagement fi#rates are first line treatment Question 5= A 12,year,old female who is 15 weeks into her first pregnancy is in0estigated for e)cessi0e sweating and tremor. Blood tests re0eal the following$ :@' N 3.36 muCl :5 1GG nmolCl What is the most appropriate management8 A. /mmediate surgery B. Car#imaIole C. @urgery at start of third trimester . 9ropylthiouracil !. "adioiodine 9ropylthiouracil is traditionally taught as the antithyroid drug of choice in pregnancy. :his approach was supported #y the 133A !ndocrine @ociety consensus guidelines. /t also has the ad0antage of #eing e)creted to a lesser e)tent than car#imaIole in #reast milk. espite this some endocrinologists use car#imaIole and the BD( states #oth drugs may #e used in pregnancy. Car#imaIole has rarely #een associated with aplasia cutis of the neonate 9regnancy$ thyroid pro#lems /n pregnancy there is an increase in the le0els of thyro)ine,#inding glo#ulin *:BG+. :his causes an increase in the le0els of total thyro)ine #ut does not affect the free thyro)ine le0el :hyroto)icosis Fntreated thyroto)icosis increases the risk of fetal loss& pre,eclampsia& maternal heart failure and premature la#our Gra0e's disease is the most common cause of thyroto)icosis in pregnancy. /t is also recognised that acti0ation of the :@' receptor #y 'CG may also occur , often termed transient gestational hyperthyroidism. 'CG le0els will fall in second and third trimester ;anagement propylthiouracil has traditionally #een the antithyroid drug of choice. :his approach was supported #y the 133A !ndocrine @ociety consensus guidelines maternal free thyro)ine le0els should #e kept in the upper third of the normal reference range to a0oid fetal hypothyroidism thyrotrophin receptor stimulating anti#odies should #e checked at ?3,?= weeks gestation , helps to determine risk of neonatal thyroid pro#lems #lock,and,replace regimes should not #e used in pregnancy radioiodine therapy is contraindicated 'ypothyroidism Eey points thyro)ine is safe during pregnancy

serum thyroid stimulating hormone measured in each trimester and =,G weeks post,partum some women re.uire an increased dose of thyro)ine during pregnancy #reast feeding is safe whilst on thyro)ine Question 5A A ?5,year,old female with a history of Addison's disease presents for re0iew in endocrinology clinic. @he is generally well #ut complains of a decrease in her li#ido. 7n e)amination there is a slight loss of pu#ic hair. What is the most likely cause8 A. Ad0erse effect of hydrocortisone therapy B. 11,hydro)ylase deficiency C. iethylstil#estrol deficiency . 7estrogen deficiency !. ehydroepiandrosterone * '!A+ deficiency ehydroepiandrosterone is the most a#undant circulating adrenal steroid. Adrenal glands are the main source of dehydroepiandrosterone in females , loss of functioning adrenal tissue as in Addison's disease may result in symptoms secondary to androgen deficiency& such as loss of li#ido. "esearch is ongoing as to whether routine replacement of '!A is #eneficial Addison's disease Autoimmune destruction of the adrenal glands is the commonest cause of hypoadrenalism in the FE& accounting for G34 of cases (eatures lethargy& weakness& anore)ia& DCJ& weight loss hyperpigmentation& 0itiligo& loss of pu#ic hair in women crisis$ collapse& shock& pyre)ia 7ther causes of hypoadrenalism 9rimary causes tu#erculosis metastases *e.g. #ronchial carcinoma+ meningococcal septicaemia *Waterhouse,(riderichsen syndrome+ '/J antiphospholipid syndrome @econdary causes pituitary disorders *e.g. tumours& irradiation& infiltration+ !)ogenous glucocorticoid therapy Question 5G Which one of the following is the most common non,iatrogenic cause of Cushing's syndrome8 A. !ctopic AC:' production B. Adrenal adenoma C. ;icronodular adrenal dysplasia . Adrenal carcinoma !. 9ituitary tumour Cushing's disease is the most common& non,iatrogenic& cause of Cushing's syndrome Cushing's syndrome$ causes AC:' dependent causes Cushing's disease *G34+$ pituitary tumour secreting AC:' producing adrenal hyperplasia ectopic AC:' production *6,134+$ e.g. small cell lung cancer AC:' independent causes iatrogenic$ steroids adrenal adenoma *6,134+ adrenal carcinoma *rare+ Carney comple)$ syndrome including cardiac my)oma micronodular adrenal dysplasia *0ery rare+ 9seudo,Cushing's mimics Cushing's often due to alcohol e)cess or se0ere depression causes false positi0e de)amethasone suppression test or 15 hr urinary free cortisol insulin stress test may #e used to differentiate Question 52 Which of the following results esta#lishes a diagnosis of dia#etes mellitus8 A. Asymptomatic patient with fasting glucose A.2 mmolCL on one occasion B. @ymptomatic patient with fasting glucose =.G mmolCL on two occasions C. Glycosuria %%% . Asymptomatic patient with random glucose 11.3 mmolCL on one occasion

!. @ymptomatic patient with random glucose 11.3 mmolCL on one occasion ia#etes diagnosis$ fasting L A.3& random L 11.1 , if asymptomatic need two readings ia#etes mellitus$ diagnosis Based on the World 'ealth 7rganisation 133= guidelines /f symptomatic fasting glucose L A.3 mmolCl random glucose L 11.1 mmolCl *or after A6g oral glucose tolerance test+ /f asymptomatic a#o0e criteria apply #ut must #e repeated on two occasions /mpaired fasting glucose and impaired glucose tolerance A fasting glucose LO =.1 #ut N A.3 mmolCl implies impaired fasting glucose */(G+ /mpaired glucose tolerance */G:+ is defined as fasting plasma glucose N A.3 mmolCl and 7G:: 1,hour 0alue LO A.G mmolCl #ut N 11.1 mmolCl ia#etes FE suggests$ '9eople with /(G should then #e offered an oral glucose tolerance test to rule out a diagnosis of dia#etes. A result #elow 11.1 mmolCl #ut a#o0e A.G mmolCl indicates that the person doesnMt ha0e dia#etes #ut does ha0e /G:.' Question 63 A 5?,year,old man is found to ha0e a phaeochromocytoma. Which anti,hypertensi0e medication should #e started first8 A. 9ropranolol B. "amipril C. Atenolol . 9heno)y#enIamine !. o)aIosin 9heno)y#enIamine is a non,selecti0e alpha,adrenoceptor antagonist and should #e started #efore a #eta,#locker is introduced :here is ongoing de#ate a#out the optimal medical management of phaeochromocytoma& with the suggestion that antihypertensi0e treatment regimes other than nonspecific alpha,#lockade are Kust as effecti0e and safe. :here are howe0er no trials to pro0ide an answer to this .uestion yet 9haeochromocytoma is a rare catecholamine secreting tumour. A#out 134 are familial and may #e associated with ;!D type //& neurofi#romatosis and 0on 'ippel,Lindau syndrome Basics #ilateral in 134 malignant in 134 e)tra,adrenal in 134 *most common site O organ of Tuckerkandl& adKacent to the #ifurcation of the aorta+ :ests 15 hr urinary collection of catecholamines @urgery is the definiti0e management. :he patient must first howe0er #e sta#iliIed with medical management$ alpha,#locker *e.g. pheno)y#enIamine+& gi0en #efore a#eta,#locker *e.g. propranolol+ Question 61 A 55,year,old man with a B;/ of ?A kgCmP1 presents to his G9. espite diet and e)ercise he has failed to lose a significant amount of weight. 'is G9 considers starting si#utramine. What is the mechanism of action of si#utramine8 A. @erotonin and noradrenaline uptake inhi#itor B. Leptin antagonist C. CB1 canna#inoid receptor antagonist . Blocks intestinal a#sorption of lipids !. 9ancreatic lipase inhi#itor 7#esity$ therapeutic options :he management of o#esity consists of step,wise approach$ conser0ati0e$ diet& e)ercise medical surgical Orlistat is a pancreatic lipase inhi#itor used in the management of o#esity. Ad0erse effects include faecal urgencyCincontinence and flatulence. D/C! ha0e defined criteria for use of orlistat. /t should only #e prescri#ed as part of an o0erall plan for managing o#esity in adults who ha0e$ B;/ of 1G kgCmP1 or more with associated risk factors& or B;/ of ?3 kgCmP1 or more continued weight loss e.g. 64 at ? months normally used for N 1 year Sibutramine

centrally acting appetite suppressant *inhi#itor of serotonin and noradrenaline uptake at hypothalamic sites that regular food intake+ ad0erse effects include$ hypertension *monitor #lood pressure and pulse during treatment+& constipation& headache& dry mouth& insomnia and anore)ia contraindicated if psychiatric illness& hypertension& /' & stroke& arrhythmias Rimonabant, a specific CB1 canna#inoid receptor antagonist& was withdrawn in 7cto#er 133G when the !uropean ;edicines Agency warned of serious psychiatric pro#lems including suicide Question 61 !ach one of the following is associated with pseudohypoparathyroidism& e)cept$ A. Low calcium le0els B. Low 9:' le0els C. @hortened 5th and 6th metacarpals . Low /Q !. @hort stature 'ypoparathyroidism 9rimary hypoparathyroidism decrease 9:' secretione.g. secondary to thyroid surgery low calcium& high phosphate treat with alfacalcidol 9seudohypoparathyroidism target cells #eing insensiti0e to 9:'due to a#normality in a G protein associated with low /Q& short stature& shortened 5th and 6th metacarpals low calcium& high phosphate& high 9:' diagnosis is made #y measuring urinary cA;9 and phosphate le0els following an infusion of 9:'. /n hypoparathyroidism this will cause an increase in #oth cA;9 and phosphate le0els. /n pseudohypoparathyroidism type / neither cA;9 nor phosphate le0els are increased whilst in pseudohypoparathyroidism type // only cA;9 rises. 9seudopseudohypoparathyroidism similar phenotype to pseudohypoparathyroidism #ut normal #iochemistry Question 6? A 6?,year,old female with a history of primary atrophic hypothyroidism is assessed two months following a change in her dose of le0othyro)ine. Which one of the following #est descri#es what the :@' should ideally #e8 A. Between 3.6 to 1.3 mFCl B. Between 3.6 to 1.6 mFCl C. Between 1.6 to 5.6 mFCl . Between 1.6 to ?.6 mFCl !. Between ?.6 to 6.6 mFCl A :@' 0alue #etween 3.6 to 1.6 mFCl is now considered prefera#le. osage changes should of course also take account of symptoms 'ypothyroidism$ management Eey points initial starting dose of le0othyro)ine should #e lower in elderly patients and those with ischaemic heart disease *e.g. 16Q 63 mcgCday+ following a change in thyro)ine dose thyroid function tests should #e checked after =,G weeks the therapeutic goal is 'normalisation' of the thyroid stimulating hormone *:@'+ le0el. As the maKority unaffected people ha0e a :@' 0alue 3.6Q1.6 mFCl it is now thought prefera#le to aim for a :@' in this range there is no e0idence to support com#ination therapy with le0othyro)ine and liothyronine @ide,effects of thyro)ine therapy hyperthyroidism$ due to o0er treatment reduced #one mineral density worsening of angina atrial fi#rillation Question 65 A 65,year,old man with type 1 dia#etes mellitus is found on annual re0iew to ha0e new 0essel formation at the optic disc. Jisual acuity in #oth eyes is not affected *=C2+. Blood pressure is 166CG5 mm'g. '#A1cG.54 What is the most appropriate management8 A. (ollow,up ophthalmoscopy in ? months B. Add aspirin C. Blood pressure control . :ight glycaemic control !. Laser therapy

:his patient has proliferati0e dia#etic retinopathy and urgent referral to an ophthalmologist for panretinal photocoagulation is indicated ia#etic retinopathy is the most common cause of #lindness in adults aged ?6,=6 years,old. 'yperglycaemia is thought to cause increased retinal #lood flow and a#normal meta#olism in the retinal 0essel walls. :his precipitates damage to endothelial cells and pericytes !ndothelial dysfunction leads to increased 0ascular permea#ility which causes the characteristic e)udates seen on fundoscopy. 9ericyte dysfunction predisposes to the formation of microaneurysms. Deo0asculiIation is thought to #e caused #y the production of growth factors in response to retinal ischaemia /n postgraduate e)ams you are most likely to #e asked a#out the characteristic features of the 0arious stagesCtypes of dia#etic retinopathy Background retinopathy microaneurysms *dots+ #lot haemorrhages *NO?+ hard e)udates 9re,proliferati0e retinopathy cotton wool spots *soft e)udatesU ischaemic ner0e fi#res+ L ? #lot haemorrhages 0enous #eadingClooping deepCdark cluster haemorrhages more common in :ype / ;& treat with laser photocoagulation 9roliferati0e retinopathy retinal neo0ascularisation , may lead to 0itrous haemorrhage fi#rous tissue forming anterior to retinal disc more common in :ype / ;& 634 #lind in 6 years ;aculopathy #ased on location rather than se0erity& anything is potentially serious hard e)udates and other '#ackground' changes on macula check 0isual acuity more common in :ype // ; Question 66 Which of the following is least recognised as a potential complication of acromegaly8 A. Colorectal cancer B. 'ypertension C. Cardiomyopathy . ia#etes mellitus !. 9ulmonary hypertension @econdary causes of pulmonary hypertension include C79 & congenital heart disease *!isenmenger's syndrome+& recurrent pulmonary em#olism& '/J and sarcoidosis /n acromegaly there is e)cess growth hormone secondary to a pituitary adenoma in o0er 264 of cases. A minority of cases are caused #y ectopic G'"' or G' production #y tumours e.g. pancreatic (eatures coarse& oily skin & large tongue& prognathism& interdental spaces spade,like hands& increase in shoe siIe features of pituitary tumour$ hypopituitarism& headaches& #itemporal hemianopia raised prolactin in 1C? of cases ,,L galactorrhoea =4 of patients ha0e ;!D,1 Complications hypertension dia#etes *L134+ cardiomyopathy colorectal cancer Question 6= A 6=,year,old female is admitted to /:F with a se0ere pneumonia. :hyroid function tests are most likely to show$ A. :@' normalU thyro)ine highU :? high B. :@' lowU thyro)ine lowU :? low C. :@' highU thyro)ine lowU :? low . :@' lowU thyro)ine highU :? high !. :@' highU thyro)ine normalU :? high /n sick euthyroid syndrome *now referred to as non,thyroidal illness+ it is often said that e0erything *:@'& thyro)ine and :?+ is low. /n some cases the :@' le0el may #e normal

Changes are re0ersi#le upon reco0ery from the systemic illness Question 6A A ?3,year,old female is diagnosed with ha0ing Gra0e's disease. What is her chance of de0eloping thyroid eye disease8 A. 1,64 B. 6,13V C. 13,164 . 16,164 !. 16,634 :hyroid eye disease affects #etween 16,634 of patients with Gra0e's disease. /t is thought to #e due to an autoimmune response against an autoantigen& possi#ly the :@' receptor& causing retro,or#ital inflammation. :he patient may #e eu,& hypo, or hyperthyroid at the time of presentation 9re0ention smoking is the most important modifia#le risk factor for the de0elopment of thyroid eye disease radioiodine treatment may increase the inflammatory symptoms seen in thyroid eye disease. /n a recent study of patients with Gra0e's disease around 164 de0eloped& or had worsening of& eye disease. 9rednisolone may help reduce the risk (eatures e)ophthalmos conKuncti0al oedema papilloedema ophthalmoplegia ;anagement steroids radiotherapy surgery Question 6G A 56,year,old man is re0iewed in the dia#etic clinic. 'is current medication is #asal #olus insulin regime com#ined with ramipril 13mg od& sim0astatin 53mg on and aspirin A6mg od. 7n e)amination #lood pressure is 1?1C=5 mm'g. :he following results are o#tained$ '#A1c A.?4 Al#umin$ Creatinine ratio 13.6 What is the most appropriate change to his treatment8 A.A@top aspirinia B.A/ncrease dose of ramiprilia C.AAdd an angiotensin,// receptor antagonistia .AAdd do)aIosinia !.A@top ramiprilia :he patient is on the ma)imum daily dose of ramipril. Gi0en the persistently raised al#umin$creatinine ratio dual #lockade with an angiotensin,// receptor antagonist is indicated ia#etic nephropathy$ management @creening all patients should #e screened annually al#umin$creatinine ratio *AC"+ in early morning specimen AC" L 1.6 O microal#uminuria ;anagement dietary protein restriction tight glycaemic control B9 control$ aim for N 116CA6 mm'g #enefits independent of #lood pressure control ha0e #een demonstrated for AC! inhi#itors and angiotensin // receptor #lockers , these may #e used alone or in com#ination control dyslipidaemia e.g. statins Question 62 An insulin stress test is most useful in the in0estigation of$ A. Glucagonoma B. /nsulinoma C. Addison's disease . 'ypopituitarism !. ia#etes mellitus /nsulin stress tests are also occasionally used to differentiate Cushing's from pseudo,Cushing /nsulin stress test Basics used in in0estigation of hypopituitarism /J insulin gi0en& G' and cortisol le0els measured with normal pituitary function G' and cortisol should rise

Contraindications epilepsy ischaemic heart disease adrenal insufficiency Question =3 !ach one of the following is a cause of nephrogenic dia#etes insipidus& e)cept$ A. 'ypercalcaemia B. emeclocycline C. 'istiocytosis < . Lithium !. 'ypokalaemia Causes of cranial / idiopathic post head inKury pituitary surgery craniopharyngiomas histiocytosis < / ;7A is the association of cranial ia#etes /nsipidus& ia#etes ;ellitus& 7ptic Atrophy and eafness *also known as Wolfram's syndrome+ Causes of nephrogenic / genetic *primary+ electrolytes$ hypercalcaemia& hypokalaemia drugs$ demeclocycline& lithium tu#ulo,interstitial disease$ o#struction& sickle,cell& pyelonephritis /n0estigation high plasma osmolarity& low urine osmolarity water depri0ation test Question =1 Which one of the following is not an indication for treating a patient with su#clinical hypothyroidism8 A. 9re0ious treatment of Gra0es' disease B. :@' L 13 C. "aised !@" . 9ositi0e thyroid autoanti#odies !. 7ther autoimmune disorder @u#clinical hypothyroidism Basics :@' raised #ut :?& :5 normal no o#0ious symptoms @ignificance risk of progressing to o0ert hypothyroidism is 1,64 per year *higher in men+ risk increased #y presence of thyroid autoanti#odies :reat if :@' L 13 thyroid autoanti#odies positi0e other autoimmune disorder pre0ious treatment of Gra0es' disease Question =1 Which one of the following features of haemochromatosis may #e re0ersi#le with treatment8 A. Cardiomyopathy B. 'ypogonadotrophic hypogonadism C. ia#etes mellitus . Arthropathy !. Li0er cirrhosis 'aemochromatosis is an autosomal recessi0e disorder of iron a#sorption and meta#olism resulting in iron accumulation. /t is caused #y inheritance of mutations in the '(! gene on #oth copies of chromosome =-. /t is often asymptomatic in early disease and initial symptoms often non,specific e.g. lethargy and arthralgia 9resenting features '#ronIe' skin pigmentation dia#etes mellitus

li0er$ stigmata of chronic li0er disease& hepatomegaly& cirrhosis& hepatocellular deposition+ cardiac failure *1nd to dilated cardiomyopathy+ hypogonadism *1nd to cirrhosis and pituitary dysfunction , hypogonadotrophic hypogonadism+ arthritis *especially of the hands+ "e0ersi#le complications Cardiomyopathy @kin pigmentation /rre0ersi#le complications Li0er cirrhosis-ia#etes mellitus 'ypogonadotrophic hypogonadism Arthropathy -there are rare cases of families with classic features of genetic haemochromatosis #ut no mutation in the '(! gene --whilst ele0ated li0er function tests and hepatomegaly may #e re0ersi#le& cirrhosis is not Question =? Which one of the following is least characteristic of Addison's disease8 A. 'ypoglycaemia B. ;eta#olic alkalosis C. 'yponatraemia . 'yperkalaemia !. 9ositi0e short AC:' test Addison's disease is associated with a meta#olic acidosis Addison's disease$ in0estigations /n a patient with suspected Addison's disease the definite in0estigation is a short AC:' test. 9lasma cortisol is measured #efore and ?3 minutes after gi0ing @ynacthen 163ug /;. Adrenal autoanti#odies such as anti,11,hydro)ylase may also #e demonstrated Associated electrolyte a#normalities hyperkalaemia hyponatraemia hypoglycaemia meta#olic acidosis Question =5 What is the most common cause of primary hyperaldosteronism8 A. 9ituitary tumour B. Adrenocortical adenoma C. Adrenal carcinoma . !ctopic secretion !. Bilateral idiopathic adrenal hyperplasia #ilateral idiopathic adrenal hyperplasia is the most common cause of primary hyperaldosteronism 9rimary hyperaldosteronism was pre0iously thought to #e most commonly caused #y an adrenal adenoma& termed Conn's syndrome. 'owe0er& recent studies ha0e shown that #ilateral idiopathic adrenal hyperplasia is the cause in up to A34 of cases. ifferentiating #etween the two is important as this determines treatment. Adrenal carcinoma is an e)tremely rare cause of primary hyperaldosteronism (eatures hypertension hypokalaemia *e.g. muscle weakness+ alkalosis /n0estigations high serum aldosterone low serum renin high,resolution C: a#domen ;anagement adrenal adenoma$ surgery #ilateral adrenocortical hyperplasia$ aldosterone antagonist e.g. spironolactone Question =6 Which one of the following features is least commonly seen in GitelmanMs syndrome8 A. 'ypokalaemia B. 'ypertension C. ;eta#olic alkalosis . 'ypocalciuria

!. 'ypomagnesaemia GitelmanMs syndrome$ normotension with hypokalaemia GitelmanMs syndrome 70er0iew defect in the thiaIide,sensiti0e Da% Cl, transporter in the distal con0oluted tu#ule (eatures hypokalaemia hypomagnesaemia hypocalciuria meta#olic alkalosis normotension Question == A 15,year,old woman is found to ha0e a #lood pressure of 1A3C133 mm'g during a routine medical check. @he is well and clinical e)amination is unremarka#le. Blood tests show$ Da% 153 mmolCl E% 1.= mmolCl Bicar#onate ?1 mmolC Frea ?.5 mmolCl Creatinine AA HmolCl Which one of the following in0estigations is most likely to #e diagnostic8 A.A"enal ultrasound B.A70ernight de)amethasone suppression test C.A"enin$aldosterone ratio .A;" angiography !.A11,hydro)ylase estimation Conn's syndrome is the likely diagnosis , a renin$aldosterone ratio would #e an appropriate first,line in0estigation 9rimary hyperaldosteronism was pre0iously thought to #e most commonly caused #y an adrenal adenoma& termed Conn's syndrome. 'owe0er& recent studies ha0e shown that #ilateral idiopathic adrenal hyperplasia is the cause in up to A34 of cases. ifferentiating #etween the two is important as this determines treatment. Adrenal carcinoma is an e)tremely rare cause of primary hyperaldosteronism (eatures hypertension hypokalaemia *e.g. muscle weakness+ alkalosis /n0estigations high serum aldosterone low serum renin high,resolution C: a#domen ;anagement adrenal adenoma$ surgery #ilateral adrenocortical hyperplasia$ aldosterone antagonist e.g. spironolactone Question =A What is the mode of inheritance of haemochromatosis8 A. Autosomal recessi0e B. <,linked dominant C. ;itochondrial inheritance . Autosomal dominant !. <,linked recessi0e 'aemochromatosis is an autosomal recessi0e disorder of iron a#sorption and meta#olism resulting in iron accumulation. /t is caused #y inheritance of mutations in the '(! gene on #oth copies of chromosome =-. /t is often asymptomatic in early disease and initial symptoms often non,specific e.g. lethargy and arthralgia 9resenting features '#ronIe' skin pigmentation dia#etes mellitus li0er$ stigmata of chronic li0er disease& hepatomegaly& cirrhosis& hepatocellular deposition+ cardiac failure *1nd to dilated cardiomyopathy+ hypogonadism *1nd to cirrhosis and pituitary dysfunction , hypogonadotrophic hypogonadism+ arthritis *especially of the hands+ "e0ersi#le complications Cardiomyopathy @kin pigmentation /rre0ersi#le complications Li0er cirrhosis-ia#etes mellitus 'ypogonadotrophic hypogonadism Arthropathy -there are rare cases of families with classic features of genetic haemochromatosis #ut no mutation in the '(! gene --whilst ele0ated li0er function tests and hepatomegaly may #e re0ersi#le& cirrhosis is not

Question =G A 52,year,old man with type 1 dia#etes mellitus is re0iewed. espite weight loss and therapy with metformin and glicaIide his last '#A1c is A.14. Which one of the following factors would suggest that the patient may #enefit from a meglitinide8 A. 7#esity B. Dot adhering to dia#etic diet C. 9ro#lems with hypoglycaemia from glicaIide . !rratic lifestyle !. !lderly and frail patients ;eglitinides , stimulate insulin release , good for erratic lifestyle ;eglitinides stimulate insulin release and are particularly useful for post,prandial hyperglycaemia or an erratic eating schedule& as patients take them shortly #efore meals ia#etes mellitus$ management of type 1 D/C! issued guidelines in 133G on the management of type 1 dia#etes mellitus. Eey points are listed #elow '#A1c the general target for patients is N =.64 howe0er& indi0idual targets should #e agreed with patients to encourage moti0ation '#A1c should #e checked e0ery 1,= months until sta#le& then = monthly Blood pressure target is N 153CG3 mm'g *or N 1?3CG3 mm'g if end,organ damage is present+ AC! inhi#itors are first,line Glucose lowering treatment metformin is first,line& followed #y a sulfonylurea meglitinides *insulin secretagogues+ should #e offered to patients with erratic lifestyles if '#A1c L A.64 consider human insulin or thiaIolidinediones where insulin is inappropriate metformin treatment should #e continued after starting insulin e)enatide should #e used only when insulin would otherwise #e started& o#esity is a pro#lem *B;/ L ?6 kgCmP1+ and the need for high dose insulin is likely. Continue only if #eneficial response occurs and is maintained *L 1.3 percentage point '#A1c reduction in = months and weight loss L 64 at 1 year+ 7ther risk factor modification aspirin to all patients L 63 years and to younger patients with other significant risk factors sim0astatin to all patients L 53 years and to younger patients with other significant risk factors if serum cholesterol target not reach consider increasing sim0astatin to G3mg on. if target still not reached consider using a more effecti0e statin *e.g. ator0astatin+ or adding eIetimi#e target total cholesterol is N 5.3 mmolCl if serum triglyceride le0els are L 5.6 mmolCl prescri#e fenofi#rate Question =2 Which one of the following is least associated with hepatocellular carcinoma8 A. 'epatitis C B. 9rimary #iliary cirrhosis C. Aflato)in . Wilson's disease !. 'aemochromatosis 'epatocellular carcinoma "isk factors hepatitis B and C cirrhosis-$ alcohol& haemochromatosis& primary #iliary cirrhosis alpha,1 antitrypsin deficiency hereditary tyrosinosis glycogen storage disease aflato)in drugs$ oral contracepti0e pill& ana#olic steroids porphyria cutanea tarda -Wilson's disease is an e)ception Question A3 A 65,year,old man with type 1 dia#etes mellitus is started on e)enatide. Which one of the following statements regarding e)enatide is incorrect8 A. :ypically results in weight loss B. ;ay #e com#ined with a sulfonylurea C. ;aKor ad0erse effect is diarrhoea . ;ay #e com#ined with a thiaIolidinedione

!. ;ust #e gi0en #y su#cutaneous inKection !)enatide causes 0omiting :he maKor ad0erse effect is nausea and 0omiting ia#etes mellitus$ GL9,1 and the new drugs A num#er of new drugs to treat dia#etes mellitus ha0e #ecome a0aila#le in recent years. ;uch research has focused around the role of glucagon,like peptide,1 *GL9,1+& a hormone released #y the small intestine in response to an oral glucose load Whilst it is well known that insulin resistance and insufficient B,cell compensation occur other effects are also seen in type 1 dia#etes mellitus *:1 ;+. /n normal physiology an oral glucose load results in a greater release of insulin than if the same load is gi0en intra0enously , this known as the incretin effect. :his effect is largely mediated #y GL9,1 and is known to #e decreased in :1 ;. /ncreasing GL9,1 le0els& either #y the administration of an analogue or inhi#iting its #reakdown& is therefore the target of two recent classes of drug Glucagon,like peptide,1 *GL9,1+ mimetics *e.g. e)enatide+ increase insulin secretion and inhi#it glucagon secretion licensed for use in :1 ; must #e gi0en #y su#cutaneous inKection may #e com#ined with metformin& a sulfonylurea or a thiaIolidinedione typically results in weight loss maKor ad0erse effect is nausea and 0omiting D/C! guidelines on the use of e)enatide should #e used only when insulin would otherwise #e started& o#esity is a pro#lem *B;/ L ?6 kgCmP1+ and the need for high dose insulin is likely continue only if #eneficial response occurs and is maintained *L 1.3 percentage point '#A1c reduction in = months and weight loss L 64 at 1 year+ ipeptidyl peptidase,5 * 99,5+ inhi#itors *e.g. 0ildagliptin& sitagliptin+ oral preparation Question A1 Which of the following statements is true regarding the pathophysiology of dia#etes mellitus8 A. Concordance #etween identical twins is higher in type 1 dia#etes mellitus than type 1 B. 9atients with type 1 dia#etes mellitus are rarely 'LA, "5 positi0e C. :ype 1 dia#etes mellitus is associated with 'LA, "? . 'aemochromatosis is an e)ample of primary dia#etes !. :ype 1 dia#etes mellitus is thought to #e inherited in an autosomal dominant fashion :ype 1 dia#etes mellitus is caused #y autoimmune destruction of the Beta,cells of the pancreas. /dentical twins show a genetic concordance of 534. /t is associated with 'LA, "? and "5. /t is inherited in a polygenic fashion :ype 1 dia#etes mellitus is thought to #e caused #y a relati0e deficiency of insulin and the phenomenon of insulin resistance. Age& o#esity and ethnicity are important aetiological factors. :here is almost 1334 concordance in identical twins and no 'LA associations. 'aemochromatosis is an e)ample of secondary dia#etes ia#etes$ pathophysiology :ype 1 ; autoimmune disease anti#odies against #eta cells of pancreas 'LA "5 L 'LA "? 0arious anti#odies such as islet,associated antigen */AA+ anti#ody and glutamic acid decar#o)ylase *GA + anti#ody are detected in patients who later go on to de0elop type 1 ; , their prognostic significance is not yet clear Question A1 A 65,year,old female presents to the !mergency epartment concerned a#out dou#le 0ision. @he is noted to ha0e e)ophthalmos and conKuncti0al oedema on e)amination and a diagnosis of thyroid eye disease is suspected. What can #e said regarding her thyroid status8 A. 'yper, or euthyroid B. 'ypothyroid C. 'yperthyroid . 'ypo, or euthyroid !. !u,& hypo, or hyperthyroid :hyroid eye disease affects #etween 16,634 of patients with Gra0e's disease. /t is thought to #e due to an autoimmune response against an autoantigen& possi#ly the :@' receptor& causing retro,or#ital inflammation. :he patient may #e eu,& hypo, or hyperthyroid at the time of presentation 9re0ention smoking is the most important modifia#le risk factor for the de0elopment of thyroid eye disease

radioiodine treatment may increase the inflammatory symptoms seen in thyroid eye disease. /n a recent study of patients with Gra0e's disease around 164 de0eloped& or had worsening of& eye disease. 9rednisolone may help reduce the risk (eatures e)ophthalmos conKuncti0al oedema papilloedema ophthalmoplegia ;anagement steroids radiotherapy surgery Question A? Which one of the following is the most common cause of hypothyroidism in the FE8 A. 9ituitary failure B. ietary iodine deficiency C. Lithium therapy . 9rimary atrophic hypothyroidism !. 'ashimoto's thyroiditis /n !uropean countries primary atrophic hypothyroidism is the most cause causes of hypothyroidism& whereas in Dorth America 'ashimoto's thyroiditis appears to account for the maKority of cases. :he reason for this discrepancy is unclear 'ypothyroidism affects around 1,14 of women in the FE and is around 6,13 times more common in females than males. 9rimary hypothyroidism 9rimary atrophic hypothyroidism most common cause autoimmune disease& associated with / ;& Addison's or pernicious anaemia 6 times more common in women 'ashimoto's thyroiditis autoimmune disease as a#o0e with goitre *positi0e microsomal anti#odies+ may cause transient thyroto)icosis in the acute phase 13 times more common in women After thyroidectomy or radioiodine treatment rug therapy *e.g. lithium& amiodarone or anti,thyroid drugs such as car#imaIole+ ietary iodine deficiency @econdary hypothyroidism *rare+ (rom pituitary failure 7ther associated conditions own's syndrome :urner's syndrome coeliac disease Question A5 ynamic pituitary function tests may #e used to assess each one of the following& e)cept$ A. Cortisol B. 9rolactin C. Growth hormone . (ollicular stimulating hormone !. Antidiuretic hormone A dynamic pituitary function test is used to assess patients with suspected primary pituitary dysfunction /nsulin& :"' and L'"' are gi0en to the patient following which the serum glucose& cortisol& growth hormone& :@'& L' and (@' le0els are recorded at regular inter0als. 9rolactin le0els are also sometimes measuredA normal dynamic pituitary function test has the following characteristics$ G' le0el rises L 13muCl cortisol le0el rises L 663 mmolCl :@' le0el rises #y L 1 muCl from #aseline le0el L' and (@' should dou#le -dopamine antagonist tests using metoclopramide may also #e used in the in0estigation of hyperprolactinaemia. A normal response is at least a twofold rise in prolactin. A #lunted prolactin response suggests a prolactinoma Question A6 A 1=,year,old o#ese female is in0estigated for menstrual distur#ance. A diagnosis of polycystic o0arian syndrome is made. Which of the following findings is most consistently seen in polycystic o0arian syndrome8 A. 7#esity B. 'irsuitism C. 70arian cysts on ultrasound

. "aised L'$ (@' ratio !. /nfertility Whilst all of the features listed a#o0e may #e seen in polycystic o0arian syndrome& o0arian cysts are the most consistent feature. 9olycystic o0ary syndrome *9C7@+ is a comple) condition of o0arian dysfunction thought to affect #etween 6,134 of women of reproducti0e age. :he aetiology of 9C7@ is not fully understood. Both hyperinsulinaemia and high le0els of luteiniIing hormone are seen in 9C7@ and there appears to #e some o0erlap with the meta#olic syndrome (eatures su#fertility and infertility menstrual distur#ances$ oligomenorrhea and amenorrhoea hirsuitism& acne *due to hyperandrogenism+ o#esity acanthosis nigricans *due to insulin resistance+ /n0estigations pel0ic ultrasound (@'& L'& prolactin& :@'& and testosterone are useful in0estigations$ raised L'$(@' ratio is a 'classical' feature #ut is no longer thought to #e useful is diagnosis. 9rolactin may #e normal or mildly ele0ated. :estosterone may #e normal or mildly ele0ated , howe0er& if markedly raised consider other causes check for impaired glucose tolerance Question A= A 52,year,old woman is in0estigated for thyroto)icosis. 7n e)amination she is noted to ha0e a goitre containing multiple irregular nodules. Duclear scintigraphy with technetium 22m re0eals patchy uptake. What is the treatment of choice8 A. Corticosteroids B. "adioiodine C. Block,and,replace regime . @urgery !. Anti,thyroid drug titration regime :o)ic multinodular goitre descri#es a thyroid gland that contains a num#er of autonomously functioning thyroid nodules that secrete e)cess thyroid hormones Duclear scintigraphy re0eals patchy uptake :he treatment of choice is radioiodine therapy Question AA Which one of the following is not part of the diagnostic criteria for the meta#olic syndrome8 A. 'igh triglycerides B. Low ' L C. 'igh L L . Central o#esity !. 'ypertension 'igh L L le0els are not part of the World 'ealth 7rganiIation or /nternational ia#etes (ederation diagnostic criteria ;eta#olic syndrome Fnfortunately there are a num#er of competing definitions of meta#olic syndrome around at the present time. /n 1222 the World 'ealth 7rganiIation produced diagnostic criteria which re.uired the presence of dia#etes mellitus& impaired glucose tolerance& impaired fasting glucose or insulin resistance& AD two of the following$ #lood pressure$ L 153C23 mm'g dyslipidaemia$ triglycerides *:G+$ L 1.=26 mmolCL andCor high,density lipoprotein cholesterol *' L,C+ N 3.2 mmolCL *male+& N 1.3 mmolCL *female+ central o#esity$ waist$hip ratio L 3.23 *male+& L 3.G6 *female+& andCor #ody mass inde) L ?3 kgCm1 microal#uminuria$ urinary al#umin e)cretion ratio L 13 mgCmin or al#umin$creatinine ratio L ?3 mgCg :he /nternational ia#etes (ederation produced a consensus set of diagnostic criteria in 1336& which are now widely in use. :hese re.uire the presence of central o#esity *defined as waist circumference L 25cm for !uropid men and L G3cm for !uropid women& with ethnicity specific 0alues for other groups+ plus any two of the following four factors$ raised :G le0el$ L 1.A mmolCL& or specific treatment for this lipid a#normality reduced ' L cholesterol$ N 1.3? mmolCL in males and N 1.12 mmolCL in females& or specific treatment for this lipid a#normality raised #lood pressure$ systolic B9 L 1?3 or diastolic B9 L G6 mm 'g& or treatment of pre0iously diagnosed hypertension raised fasting plasma glucose *(9G+ L 6.= mmolCL& or pre0iously diagnosed type 1 dia#etes. /f a#o0e 6.= mmolCL& 7G:: is strongly recommended #ut is not necessary to define presence of the syndrome. 7ther associated features include$ raised uric acid le0els

polycystic o0arian syndrome Question AG A ?6,year,old female is referred to the endocrine clinic due to weight loss and palpitations. :he following results are o#tained$ :@' N 3.36 muCl :5 1AG mmolCl Which one of the following features would most suggest a diagnosis of Gra0e's disease8 A. Atrial fi#rillation B. Lid lag C. (amily history of radioiodine treatment . 9reti#ial my)oedema !. ;ultinodular goitre 9reti#ial my)oedema is not seen in other causes of thyroto)icosis and points towards a diagnosis of Gra0e's (eatures seen in Gra0e's #ut not in other causes of thyroto)icosis eye signs$ e)ophthalmos& ophthalmoplegia preti#ial my)oedema thyroid acropachy Autoanti#odies anti,:@' receptor stimulating anti#odies *234+ anti,thyroid pero)idase anti#odies *634+ Question A2 An 1G,year,old male de0elops type 1 dia#etes mellitus. Which one of the following genes is most likely to #e responsi#le8 A. Glucokinase B. 'D(,1 alpha C. 'D(,5 alpha . 'D(,1 #eta !. /9(,1 ;aturity,onset dia#etes of the young *;7 >+ is characterised #y the de0elopment of type 1 dia#etes mellitus in patients N 16 years old. /t is typically inherited as an autosomal dominant condition. 70er si) different genetic mutations ha0e so far #een identified as leading to ;7 >. Eetosis is not a feature at presentation ;7 > ? =34 of cases due to a defect in the 'D(,1 alpha gene ;7 > 1 134 of cases due to a defect in the glucokinase gene Question G3 !ach one of the following is a cause of nephrogenic dia#etes insipidus& e)cept$ A. 'ypocalcaemia B. @ickle,cell anaemia C. Lithium . 'ypokalaemia !. emeclocycline Causes of cranial / idiopathic post head inKury pituitary surgery craniopharyngiomas histiocytosis < / ;7A is the association of cranial ia#etes /nsipidus& ia#etes ;ellitus& 7ptic Atrophy and eafness *also known as Wolfram's syndrome+ Causes of nephrogenic / genetic *primary+ electrolytes$ hypercalcaemia& hypokalaemia drugs$ demeclocycline& lithium tu#ulo,interstitial disease$ o#struction& sickle,cell& pyelonephritis /n0estigation high plasma osmolarity& low urine osmolarity water depri0ation test Question G1

A =1,year,old man is in0estigated for hypertension and pro)imal myopathy. 7n e)amination he is noted to ha0e a#dominal striae. Which one of the following is most associated with ectopic AC:' secretion8 A. Carcinoid tumour B. @mall cell lung cancer C. Cardiac my)oma . @.uamous cell lung cancer !. Adrenal carcinoma @mall cell lung cancer accounts 63,A64 of case of ectopic AC:' Adrenal carcinoma and cardiac my)oma are causes of AC:' independent Cushing's syndrome Cushing's syndrome$ causes AC:' dependent causes Cushing's disease *G34+$ pituitary tumour secreting AC:' producing adrenal hyperplasia ectopic AC:' production *6,134+$ e.g. small cell lung cancer AC:' independent causes iatrogenic$ steroids adrenal adenoma *6,134+ adrenal carcinoma *rare+ Carney comple)$ syndrome including cardiac my)oma micronodular adrenal dysplasia *0ery rare+ 9seudo,Cushing's mimics Cushing's often due to alcohol e)cess or se0ere depression causes false positi0e de)amethasone suppression test or 15 hr urinary free cortisol insulin stress test may #e used to differentiate Question G1 Cushing's syndrome is most typically associated with which one of the following a#normalities$ A. 'ypokalaemic meta#olic acidosis B. 'yperkalaemic meta#olic alkalosis C. 'ypocalcaemic meta#olic acidosis . 'ypokalaemic meta#olic alkalosis !. 'yperkalaemic meta#olic acidosis Cushing's syndrome$ in0estigations /n0estigations are di0ided into confirming Cushing's syndrome and then localising the lesion. A hypokalaemic meta#olic alkalosis may #e seen& along with impaired glucose tolerance. !ctopic AC:' secretion *e.g. secondary to small cell lung cancer+ is characteristically associated with 0ery low potassium le0els. An insulin stress test is used to differentiate #etween true Cushing's and pseudo,Cushing's :ests to confirm Cushing's syndrome :he two most commonly used tests are$ o0ernight de)amethasone suppression test *most sensiti0e+ 15 hr urinary free cortisol Localisation tests :he first,line localisation is 2am and midnight plasma AC:' *and cortisol+ le0els. /f AC:' is suppressed then a non, AC:' dependent cause is likely such as an adrenal adenoma 'igh,dose de)amethasone suppression test if pituitary source then cortisol suppressed if ectopicCadrenal then no change in cortisol C"' stimulation if pituitary source then cortisol rises if ectopicCadrenal then no change in cortisol 9etrosal sinus sampling of AC:' may #e needed to differentiate #etween pituitary and ectopic AC:' secretion Question G? Which one of the following statements regarding the management of dia#etes mellitus during pregnancy is incorrect8 A. A pre0ious macrosomic #a#y is a risk factor for gestational dia#etes B. ia#etes complicates around 1 in 53 pregnancies C. A higher dose of folic acid *6 mgCday+ should #e used . ;etformin is contraindicated !. :ight glycaemic control reduces complication rates :here is increasing e0idence that metformin is safe during pregnancy 9regnancy$ dia#etes mellitus

ia#etes mellitus may #e a pre,e)isting pro#lem or de0elop during pregnancy& gestational dia#etes. /t complicates around 1 in 53 pregnancies "isk factors for gestational dia#etes B;/ of L ?3 kgCmP1 pre0ious macrosomic #a#y weighing 5.6 kg or a#o0e. pre0ious gestational dia#etes first,degree relati0e with dia#etes family origin with a high pre0alence of dia#etes *@outh Asian& #lack Cari##ean and ;iddle !astern+ @creening for gestational dia#etes , if a women has had gestational dia#etes pre0iously an oral glucose tolerance test *7G::+ should #e performed at 1=,1G weeks and at 1G weeks if the first test is normal women with any of the other risk factors should #e offered an 7G:: at 15Q1G weeks D/C! issued guidelines on the management of dia#etes mellitus in pregnancy in 133G ;anagement weight loss for women with B;/ of L 1A kgCmP1 stop oral hypoglycaemic agents& apart from metformin-& and commence insulin folic acid 6 mgCday from pre,conception to 11 weeks gestation detailed anomaly scan at 1G,13 weeks including four,cham#er 0iew of the heart and outflow tracts tight glycaemic control reduces complication rates treat retinopathy as can worsen during pregnancy Women who de0elop gestational dia#etes should stop taking hypoglycaemic medication following deli0ery. A fasting glucose should #e checked at the = week postnatal check -there is increasing e0idence that metformin is safe during pregnancy Question G5 A 52,year,old woman with type 1 dia#etes mellitus is #eing considered for e)enatide therapy. Which one of the following is not part of the D/C! criteria for starting or continuing this drug8 A. B;/ L ?6 kgCmP1 B. Greater than 1.3 percentage point '#A1c reduction after = months C. 'as failed with insulin therapy . 'as type 1 dia#etes mellitus !. Weight loss L 64 at 1 year 9atients do not need to ha0e #een on insulin prior to using e)enatide ia#etes mellitus$ GL9,1 and the new drugs A num#er of new drugs to treat dia#etes mellitus ha0e #ecome a0aila#le in recent years. ;uch research has focused around the role of glucagon,like peptide,1 *GL9,1+& a hormone released #y the small intestine in response to an oral glucose load Whilst it is well known that insulin resistance and insufficient B,cell compensation occur other effects are also seen in type 1 dia#etes mellitus *:1 ;+. /n normal physiology an oral glucose load results in a greater release of insulin than if the same load is gi0en intra0enously , this known as the incretin effect. :his effect is largely mediated #y GL9,1 and is known to #e decreased in :1 ;. /ncreasing GL9,1 le0els& either #y the administration of an analogue or inhi#iting its #reakdown& is therefore the target of two recent classes of drug Glucagon,like peptide,1 *GL9,1+ mimetics *e.g. e)enatide+ increase insulin secretion and inhi#it glucagon secretion licensed for use in :1 ; must #e gi0en #y su#cutaneous inKection may #e com#ined with metformin& a sulfonylurea or a thiaIolidinedione typically results in weight loss maKor ad0erse effect is nausea and 0omiting D/C! guidelines on the use of e)enatide should #e used only when insulin would otherwise #e started& o#esity is a pro#lem *B;/ L ?6 kgCmP1+ and the need for high dose insulin is likely continue only if #eneficial response occurs and is maintained *L 1.3 percentage point '#A1c reduction in = months and weight loss L 64 at 1 year+ ipeptidyl peptidase,5 * 99,5+ inhi#itors *e.g. 0ildagliptin& sitagliptin+ oral preparation Question G6 A =2,year,old man who had a stroke = months ago is re0iewed. After his diagnosis he was started on sim0astatin 53mg on for secondary pre0ention of further cardio0ascular disease. A fasting lipid profile taken one week ago is reported as follows$ :otal cholesterol 5.5 mmolCl ' L cholesterol 1.3 mmolCl L L cholesterol ?.1 mmolCl :riglyceride1.6 mmolCl According to recent D/C! guidelines& what is the most appropriate action8 A. @witch to sim0astatin G3mg on B. Do change in medication& repeat lipid profile in = months

C. Add nicotinic acid . @witch to ator0astatin G3mg on !. Add eIetimi#e Any change is of course in conKunction with continued lifestyle ad0ice. 'yperlipidaemia$ management /n 133G D/C! issued guidelines on lipid modification. Eey points are summarised #elow. 9rimary pre0ention A systematic strategy should #e used to identify people aged 53QA5 who are likely to #e at high risk of cardio0ascular disease *CJ +& defined as a 13,year risk of 134 or greater. :he 1221 (ramingham e.uations are still recommended to assess 13,year CJ risk. /t is howe0er recommended that adKustments are made in the following situations$ first,degree relati0e with a history of premature coronary heart disease *defined as N 66 years in males and N =6 years in females+ , increase risk #y 1.6 times if one relati0e affected or up to 1.3 times if more than one relati0e affected @outh Asian ethnicity , increase risk #y 1.5 times Along with lifestyle changes drug treatment should #e considered for patients with a 13,year CJ risk of 134 or greater sim0astatin 53mg on is the first line treatment there is no target le0el for total or L L cholesterol for primary pre0ention li0er function tests should #e check at #aseline& within ? months and at 11 months #ut not again unless clinically indicated @econdary pre0ention All patients with CJ should #e taking a statin in the a#sence of any contraindication D/C! recommend increasing to sim0astatin G3 mg if a total cholesterol of less than 5 mmolClitre or an L L cholesterol of less than 1 mmolClitre is not attained Question G= Which one of the following statements regarding D/C! guidance on the primary pre0ention of cardio0ascular disease is incorrect8 A. 9remature coronary heart disease is defined as N =6 years in females B. A 13,year risk of 164 is used to identify patients who should #e considered for lipid,lowering therapy C. @im0astatin 53mg on is the first line treatment in patients with a significant risk . :he 1221 (ramingham e.uations are still recommended for calculating risk !. /f a patient has a first degree relati0e with premature heart disease the risk should #e multiplied #y 1.6 9rimary pre0ention CJ $ 13,year risk of 134 is cut,off 'yperlipidaemia$ management /n 133G D/C! issued guidelines on lipid modification. Eey points are summarised #elow. 9rimary pre0ention A systematic strategy should #e used to identify people aged 53QA5 who are likely to #e at high risk of cardio0ascular disease *CJ +& defined as a 13,year risk of 134 or greater. :he 1221 (ramingham e.uations are still recommended to assess 13,year CJ risk. /t is howe0er recommended that adKustments are made in the following situations$ first,degree relati0e with a history of premature coronary heart disease *defined as N 66 years in males and N =6 years in females+ , increase risk #y 1.6 times if one relati0e affected or up to 1.3 times if more than one relati0e affected @outh Asian ethnicity , increase risk #y 1.5 times Along with lifestyle changes drug treatment should #e considered for patients with a 13,year CJ risk of 134 or greater sim0astatin 53mg on is the first line treatment there is no target le0el for total or L L cholesterol for primary pre0ention li0er function tests should #e check at #aseline& within ? months and at 11 months #ut not again unless clinically indicated @econdary pre0ention All patients with CJ should #e taking a statin in the a#sence of any contraindication D/C! recommend increasing to sim0astatin G3 mg if total cholesterol of less than 5 mmolClitre or an L L cholesterol of less than 1 mmolClitre is not attained Question GA Which one of the following is least associated with gynaecomastia8 A. Elinefelter's syndrome B. @eminoma C. Li0er disease . 9u#erty !. 'ypothyroidism Gynaecomastia is seen in up to a third of men with thyroto)icosis& #ut is not a feature of hypothyroidism Gynaecomastia descri#es an a#normal amount of #reast tissue in males and is usually caused #y an increased oestrogen$androgen ratio. /t is important to differentiate the causes of galactorrhoea *due to the actions of prolactin on #reast tissue+ from those of gynaecomastia Causes of gynaecomastia

physiological$ normal in pu#erty syndromes with androgen deficiency$ Eallman's& Elinefelter's testicular failure$ e.g. mumps li0er disease testicular cancer e.g. seminoma secreting hCG ectopic tumour secretion hyperthyroidism haemodialysis drugs$ see #elow rug causes of gynaecomastia spironolactone *most common drug cause+ cimetidine digo)in canna#is oestrogens& ana#olic steroids Jery rare drug causes of gynaecomastia tricyclics isoniaIid calcium channel #lockers heroin #usulfan methyldopa Question GG Which one of the following regarding the management of thyroid pro#lems during pregnancy is incorrect8 A. ;aternal free thyro)ine le0els should #e kept in the upper third of the normal reference range B. /ncreased le0els of thyro)ine,#inding glo#ulin are seen in pregnancy C. Block,and,replace is prefera#le in pregnancy compared to antithyroid drug titration . Breast feeding is safe whilst on thyro)ine !. Fntreated thyroto)icosis increases the risk of pre,eclampsia 9regnancy$ thyroid pro#lems /n pregnancy there is an increase in the le0els of thyro)ine,#inding glo#ulin *:BG+. :his causes an increase in the le0els of total thyro)ine #ut does not affect the free thyro)ine le0el :hyroto)icosis Fntreated thyroto)icosis increases the risk of fetal loss& pre,eclampsia& maternal heart failure and premature la#our Gra0e's disease is the most common cause of thyroto)icosis in pregnancy. /t is also recognised that acti0ation of the :@' receptor #y 'CG may also occur , often termed transient gestational hyperthyroidism. 'CG le0els will fall in second and third trimester ;anagement propylthiouracil has traditionally #een the antithyroid drug of choice. :his approach was supported #y the 133A !ndocrine @ociety consensus guidelines maternal free thyro)ine le0els should #e kept in the upper third of the normal reference range to a0oid fetal hypothyroidism thyrotrophin receptor stimulating anti#odies should #e checked at ?3,?= weeks gestation , helps to determine risk of neonatal thyroid pro#lems #lock,and,replace regimes should not #e used in pregnancy radioiodine therapy is contraindicated 'ypothyroidism Eey points thyro)ine is safe during pregnancy serum thyroid stimulating hormone measured in each trimester and =,G weeks post,partum some women re.uire an increased dose of thyro)ine during pregnancy #reast feeding is safe whilst on thyro)ine Question G2 A 1?,year,old woman is diagnosed with Gra0e's disease. Which one of the following statements regarding treatment is correct8 A. Block,and,replace regimes are usually of a shorter duration than car#imaIole titration therapy B. Concurrent administration of propranolol and car#imaIole should #e a0oided C. 9atients on #lock,and,replace regimes ha0e fewer side,effects than those using titration therapy . Car#imaIole should #e started at no higher than 13mgCday for patients commencing a titration regime !. /n the #lock,and,replace regime le0othyro)ine should #e started at the same time as car#imaIole Gra0e's disease$ management

espite many trials there is no clear guidance on the optimal management of Gra0e's disease. :reatment options include titration of anti,thyroid drugs *A: s& for e)ample car#imaIole+& #lock,and,replace regimes& radioiodine treatment and surgery. 9ropranolol is often gi0en initially to #lock adrenergic effects A: titration car#imaIole is started at 53mg and reduced gradually to maintain euthyroidism typically continued for 11,1G months patients following an A: titration regime ha0e #een shown to suffer fewer side,effects than those on a #lock,and,replace regime Block,and,replace car#imaIole is started at 53mg thyro)ine is added when the patient is euthyroid treatment typically lasts for =,2 months :he maKor complication of car#imaIole therapy is agranulocytosis "adioiodine treatment contraindications include pregnancy *should #e a0oided for 5,= months following treatment+ and age N 1= years. :hyroid eye disease is a relati0e contraindication& as it may worsen the condition the proportion of patients who #ecome hypothyroid depends on the dose gi0en& #ut as a rule the maKority of patient will re.uire thyro)ine supplementation after 6 years Question 23 A 65,year,old man with type 1 dia#etes mellitus is re0iewed in clinic. 'e is currently taking rosiglitaIone& aspirin and sim0astatin. Which one of the following pro#lems is most likely caused #y rosiglitaIone8 A. 9hotosensiti0ity B. :hrom#ocytopaenia C. ;yalgia . 9eripheral oedema !. 'yponatraemia :hiaIolidinediones are a new class of agents used in the treatment of type 1 dia#etes mellitus. :hey are agonists to the 99A",gamma receptor and reduce peripheral insulin resistance :he 99A",gamma receptor is an intracellular nuclear receptor. /ts natural ligands are free fatty acids and it is thought to control adipocyte differentiation and function Ad0erse effects weight gain li0er impairment$ monitor L(:s fluid retention , therefore contraindicated in heart failure recent studies ha0e indicated an increased risk of fractures rosiglitaIone is not recommended for use in patients with ischaemic heart disease or peripheral arterial disease. :he risk of complications may #e increased if rosiglitIone is com#ined with insulin Question 21 A 66,year,old woman with type 1 dia#etes mellitus is re0iewed. A decision is made to start thiaIolidinedione therapy. Which one of the following points is it most rele0ant to consider #efore starting treatment8 A. 'istory of oesophageal pro#lems B. (racture risk C. 'istory of depression . 'istory of cardiac arrhythmias !. Jisual acuity :here is increasing e0idence thiaIolidinediones increase the risk of fractures :hiaIolidinediones are a new class of agents used in the treatment of type 1 dia#etes mellitus. :hey are agonists to the 99A",gamma receptor and reduce peripheral insulin resistance :he 99A",gamma receptor is an intracellular nuclear receptor. /ts natural ligands are free fatty acids and it is thought to control adipocyte differentiation and function Ad0erse effects weight gain li0er impairment$ monitor L(:s fluid retention , therefore contraindicated in heart failure recent studies ha0e indicated an increased risk of fractures rosiglitaIone is not recommended for use in patients with ischaemic heart disease or peripheral arterial disease. :he risk of complications may #e increased if rosiglitIone is com#ined with insulin Question 21

A 6?,year,old hea0y goods 0ehicle dri0er with a history of type // dia#etes mellitus is re0iewed in the dia#etes clinic. espite ma)imal oral hypoglycaemic therapy his '#A1c is 2.A4. /f insulin therapy is started then which one of the following is most appropriate with regards to his Ko#8 A. Cannot continue to dri0e hea0y goods 0ehicle B. /nform JLA and recommence dri0ing once sta#le insulin dose achie0ed C. Can only dri0e during daylight hours . As under 66 years of age then no re.uirement to inform JLA !. Deeds annual screening to e)clude retinopathy or neuropathy 9atients on insulin cannot hold a 'GJ licence JLA$ dia#etes mellitus :he guidelines #elow relate to carCmotorcycle use unless specifically stated. (or o#0ious reasons& the rules relating to dri0ers of hea0y goods 0ehicles *'GJs+ tend to #e much stricter @pecific rules if on insulin then cannot hold 'GJ licenceif on insulin then patient can dri0e a car as long as they ha0e hypoglycaemic awareness and no rele0ant 0isual impairment if diet controlled alone and no rele0ant complications *e.g. maculopathy+ then no re.uirement to inform JLA -there are comple) e)ceptions to this rule& #ut these are not rele0ant for the purposes of the e)am Question 2? What is the most appropriate screening in0estigation to e)clude a phaeochromocytoma8 A. Fltrasound adrenals B. 9heno)y#enIamine suppression test C. 15 hour urinary collection of 0anillylmandelic acid . 15 hour urinary collection of catecholamines !. 9lasma adrenaline *morning+ 9haeochromocytoma$ do 15 hr urinary catecholamines& not J;A etc A 15 hr urinary collection of catecholamines is preferred to one of 0anillylmandelic acid as it has a higher sensiti0ity. :hree 15 hour collections are needed as some patients ha0e intermittently raised le0els 9haeochromocytoma is a rare catecholamine secreting tumour. A#out 134 are familial and may #e associated with ;!D type //& neurofi#romatosis and 0on 'ippel,Lindau syndrome Basics #ilateral in 134 malignant in 134 e)tra,adrenal in 134 *most common site O organ of Tuckerkandl& adKacent to the #ifurcation of the aorta+ :ests 15 hr urinary collection of catecholamines @urgery is the definiti0e management. :he patient must first howe0er #e sta#iliIed with medical management$ alpha,#locker *e.g. pheno)y#enIamine+& gi0en #efore a #eta,#locker *e.g. propranolol+ Question 25 !ach one of the following is associated with 9endred's syndrome& e)cept$ A. Goitre B. @hort 5th and 6th metacarpals C. Autosomal recessi0e inheritance . @ensorineural deafness !. !uthyroid status 9endred's syndrome Autosomal recessi0e disorder of defecti0e iodine uptake (eatures sensorineural deafness goitre euthyroid or mild hypothyroidism Question 26 Which one of the following conditions may cause hypokalaemia in association with hypertension8 A. Gitelman syndrome B. 11,hydro)ylase deficiency C. Bartter's syndrome

. 9haeochromocytoma !. 11,#eta hydro)ylase deficiency 11,hydro)ylase deficiency& which accounts for 234 of congenital adrenal hyperplasia cases& is not associated with hypertension 'ypokalaemia and hypertension (or postgraduate e)ams it is useful to #e a#le to classify the causes of hypokalaemia in to those associated with hypertension& and those which are not 'ypokalaemia with hypertension Cushing's syndrome Conn's syndrome *primary hyperaldosteronism+ Liddle's syndrome 11,#eta hydro)ylase deficiencyCar#eno)olone& an anti,ulcer drug& and li.uorice e)cess can potentially cause hypokalaemia associated with hypertension 'ypokalaemia without hypertension diuretics G/ loss *e.g. diarrhoea& 0omiting+ renal tu#ular acidosis *type 1 and 1--+ Bartter's syndrome Gitelman syndrome -11,hydro)ylase deficiency& which accounts for 234 of congenital adrenal hyperplasia cases& is not associated with hypertension --type 5 renal tu#ular acidosis is associated with hyperkalaemia Question 2= A ?3,year,old female is started on car#imaIole 13mg #d following a diagnosis of Gra0e's disease. What is the #est #iochemical marker to assess her response to treatment8 A. :otal :5 B. :@' C. (ree :5 . !@" !. (ree :? :he answer the College are looking for is :@'. :here is howe0er a significant proportion of patients for whom :@' monitoring alone is insufficient. :@' may remain suppressed for se0eral weeks as continued production of thyroid stimulating immunoglo#ulins seen in Gra0e's disease reduces the need for the pituitary to secrete :@' :hyroto)icosis Causes Gra0e's disease to)ic nodule goitres su#acute *de Quer0ain's+ thyroiditis post,partum thyroiditis acute phase of 'ashimoto's thyroiditis *later results in hypothyroidism+ to)ic adenoma *9lummer's disease+ amiodarone therapy /n0estigation :@' down& :5 and :? up thyroid autoanti#odies other in0estigations are not routinely done #ut includes isotope scanning Question 2A Which of the following secondary causes of hyperlipidaemia result in predominantly hypercholesterolaemia8 A. 'ypothyroidism B. 7#esity C. Li0er disease . BendrofluaIide !. Chronic renal failure 'ypercholesterolaemia rather than hypertriglyceridaemia$ nephrotic syndrome& cholestasis& hypothyroidism 'yperlipidaemia$ secondary causes Causes of predominantly hypertriglyceridaemia dia#etes mellitus *types 1 and 1+ o#esity alcohol chronic renal failure drugs$ thiaIides& non,selecti0e #eta,#lockers& unopposed oestrogen

li0er disease Causes of predominantly hypercholesterolaemia nephrotic syndrome cholestasis hypothyroidism Question 2G A 61,year,old woman with suspected dia#etes mellitus has an oral glucose tolerance test& following the standard W'7 protocol. :he following results are o#tained$ :ime *hours+ Blood glucose *mmolCl+ 3 6.2 1 G.5 'ow should these results #e interpreted8 A. /mpaired fasting glucose and impaired glucose tolerance B. Dormal C. ia#etes mellitus . /mpaired glucose tolerance !. /mpaired fasting glucose ia#etes mellitus$ diagnosis Based on the World 'ealth 7rganisation 133= guidelines /f symptomatic fasting glucose L A.3 mmolCl random glucose L 11.1 mmolCl *or after A6g oral glucose tolerance test+ /f asymptomatic a#o0e criteria apply #ut must #e repeated on two occasions /mpaired fasting glucose and impaired glucose tolerance A fasting glucose LO =.1 #ut N A.3 mmolCl implies impaired fasting glucose */(G+ /mpaired glucose tolerance */G:+ is defined as fasting plasma glucose N A.3 mmolCl and 7G:: 1,hour 0alue LO A.G mmolCl #ut N 11.1 mmolCl ia#etes FE suggests$ '9eople with /(G should then #e offered an oral glucose tolerance test to rule out a diagnosis of dia#etes. A result #elow 11.1 mmolCl #ut a#o0e A.G mmolCl indicates that the person doesnMt ha0e dia#etes #ut does ha0e /G:.' Question 22 A =6,year,old man known to ha0e a carcinoid tumour of the appendi) is found to ha0e hepatic metastases. /f the patient de0elops carcinoid syndrome& which one of the following symptoms is most likely to occur first8 A. (acial flushing B. 'eadache C. Jomiting . iarrhoea !. 9alpitations Carcinoid tumours Carcinoid syndrome usually occurs when metastases are present in the li0er and release serotonin into the systemic circulation may also occur with lung carcinoid as mediators are not 'cleared' #y the li0er (eatures flushing *often earliest symptom+ diarrhoea #ronchospasm hypotension right heart 0al0ular stenosis *left heart can #e affected in #ronchial carcinoid+ other molecules such as AC:' and G'"' may also #e secreted resulting in& for e)ample& Cushing's syndrome pellagra can rarely de0elop as dietary tryptophan is di0erted to serotonin #y the tumour /n0estigation urinary 6,'/AA plasma chromogranin A y ;anagement somatostatin analogues e.g. octreotide diarrhoea$ cyproheptadine may help Question 133 A 16,year,old man with a family history of multiple endocrine neoplasia type 1 is re0iewed in clinic. What is the single most useful in0estigation to monitor such patients8 A.A@hort synacthen test B.AFrinary catecholamines

C.A@erum calcium .A:hyroid function tests !.A@erum prolactin :he high incidence of parathyroid tumours and hypercalcaemia make serum calcium a useful indicator of ;!D type 1 in suspected indi0iduals ;ultiple endocrine neoplasia the three main types of multiple endocrine neoplasia *;!D+ is inherited as an autosomal dominant disorder MEN type I ;nemonic 'three 9's'$ W parathyroid *264+$ hyperparathyroidism due to parathyroid hyperplasia W pituitary *A34+ W pancreas *634& e.g. gastrinoma+ W also$ adrenal and thyroid -;!D1 gene -;ost common presentation O hypercalcaemia MEN type IIa W phaeochromocytoma *264& e.g. phaeochromocytoma+ W medullary thyroid cancer *A34+ W parathyroid *=34+ -"!: oncogene MEN type IIb W medullary thyroid cancer W phaeochromocytoma W marfanoid #ody ha#itus W neuromas -"!: oncogene Question 131 !ach one of the following is associated with autoimmune polyendocrinopathy syndrome type 1& e)cept$ A. Chronic mucocutaneous candidiasis B. Addison's disease C. 9rimary hyperparathyroidism . Autosomal recessi0e inheritance !. A mutation of the A/"!1 gene on chromosome 11 Autoimmune polyendocrinopathy syndrome Addison's disease *autoimmune hypoadrenalism+ is associated with other endocrine deficiencies in appro)imately 134 of patients. :here are two distinct types of autoimmune polyendocrinopathy syndrome *A9@+& with type 1 *sometimes referred to as @chmidt's syndrome+ #eing much more common. A9@ type 1 has a polygenic inheritance and is linked to 'LA "?C "5. 9atients ha0e Addison's disease plus either$ type 1 dia#etes mellitus autoimmune thyroid disease A9@ type 1 is occasionally referred to as ;ultiple !ndocrine eficiency Autoimmune Candidiasis *;! AC+. /t is a 0ery rare autosomal recessi0e disorder caused #y mutation of A/"!1 gene on chromosome 11 (eatures of A9@ type 1 *1 out of ? needed+ chronic mucocutaneous candidiasis *typically first feature as young child+ Addison's disease primary hypoparathyroidism Jitiligo can occur in #oth types Question 131 Which one of the following features is least associated with primary hyperparathyroidism8 A. epression B. 9olydipsia C. @ensory loss . 9eptic ulceration !. 'ypertension 9rimary hyperparathyroidism /n postgraduate e)ams primary hyperparathyroidism is stereotypically seen in elderly females with an un.uencha#le thirst and an inappropriately normal or raised parathyroid hormone le0el. /t is most commonly due to a solitary adenoma Causes of primary hyperparathyroidism G34$ solitary adenoma

164$ hyperplasia 54$ multiple adenoma 14$ carcinoma (eatures , '#ones& stones& a#dominal groans and psychic moans' polydipsia& polyuria peptic ulcerationCconstipationCpancreatitis #one painCfracture renal stones depression hypertension Associations hypertension multiple endocrine neoplasia$ ;!D / and // /n0estigations raised calcium& low phosphate 9:' may #e raised or normal technetium,;/B/ su#traction scan :reatment total parathyroidectomy Question 13? A 66,year,old female is re0iewed in the dia#etes clinic. :he following results are o#tained$ Frinalysis protein % '#A1c 13.34 What a0erage #lood glucose le0el for the past 1 months is this most likely to represent8 A. 2 B. 13 C. 11 . 16 !. :here is no relation #etween '#A1c and a0erage #lood glucose Glycosylated haemoglo#in *'#A1c+ is the most widely used measure of long,term glycaemic control in dia#etes mellitus. '#A1c is produced #y the glycosylation of haemoglo#in at a rate proportional to the glucose concentration. :he le0el of '#A1c therefore is dependant on red #lood cell lifespan a0erage #lood glucose concentration '#A1c is generally thought to reflect the #lood glucose o0er the pre0ious '1,? months' although there is some e0idence it is weighed more strongly to glucose le0els of the past 1,5 weeks :he relationship #etween '#A1c and a0erage #lood glucose is comple) #ut has #een studied #y the ia#etes Control and Complications :rial 'BA1c *4+ A0erage plasma glucose *mmolCl+ 6 6.6 = A.6 A 2.6 G 11.6 2 1?.6 13 16.6 11 1A.6 11 12.6 (rom the a#o0e we can see that a0erage plasma glucose O *1 - '#A1c+ , 5.6 Question 135 A 65,year,old man has a routine medical for work. 'e is asymptomatic and clinical e)amination is unremarka#le. Which of the following results esta#lishes a diagnosis of impaired fasting glucose8 A. (asting glucose A.1 mmolCL on one occasion B. (asting glucose =.G mmolCL on two occasions C. Glycosuria %% . A6g oral glucose tolerance test 1 hour 0alue of G.5 mmolCL !. '#A1c of =.A4 ia#etes diagnosis$ fasting L A.3& random L 11.1 , if asymptomatic need two readings A A6g oral glucose tolerance test 1 hour 0alue of G.5 mmolCL would imply impaired glucose tolerance rather than impaired fasting glucose ia#etes mellitus$ diagnosis Based on the World 'ealth 7rganisation 133= guidelines

/f symptomatic fasting glucose LO A.3 mmolCl random glucose LO 11.1 mmolCl *or after A6g oral glucose tolerance test+ /f asymptomatic a#o0e criteria apply #ut must #e repeated on two occasions /mpaired fasting glucose and impaired glucose tolerance A fasting glucose LO =.1 #ut N A.3 mmolCl implies impaired fasting glucose */(G+ /mpaired glucose tolerance */G:+ is defined as fasting plasma glucose N A.3 mmolCl and 7G:: 1,hour 0alue LO A.G mmolCl #ut N 11.1 mmolCl ia#etes FE suggests$ '9eople with /(G should then #e offered an oral glucose tolerance test to rule out a diagnosis of dia#etes. A result #elow 11.1 mmolCl #ut a#o0e A.G mmolCl indicates that the person doesnMt ha0e dia#etes #ut does ha0e /G:.' Question 136 Which of the following is most likely to cause hypokalaemia associated with alkalosis8 A. AcetaIolamide B. 9artially treated dia#etic ketoacidosis C. iarrhoea . Cushing's syndrome !. "enal tu#ular acidosis 'ypokalaemia and acid,#ase #alance 9otassium and hydrogen can #e thought of as competitors. 'yperkalaemia tends to #e associated with acidosis #ecause as potassium le0els rise fewer hydrogen ions can enter the cells 'ypokalaemia with alkalosis 0omiting diuretics Cushing's syndrome Conn's syndrome *primary hyperaldosteronism+ 'ypokalaemia with acidosis diarrhoea renal tu#ular acidosis acetaIolamide partially treated dia#etic ketoacidosis Question 13= What is the most common hormone secreted from pituitary tumours8 A. :@' B. L' C. G' . 9rolactin !. AC:' 9ituitary tumours 'ormones secreted 9"L , ?64 no o#0ious hormone& 'non,functioning'& 'chromopho#e' , ?34 G' , 134 9"L and G' , A4 AC:' , A4 others$ :@'& L'& (@' , 14 Question 13A Which one of the following is not a feature of polycystic o0arian syndrome8 A. /mpaired glucose tolerance B. @u#fertility C. Acne . Acanthosis nigricans !. "aised (@'CL' ratio A raised L' to (@' ratio& rather than raised (@' to L' ratio& is a classic te)t#ook feature of polycystic o0arian syndrome. /t is howe0er no longer thought to #e a useful diagnostic marker& #ut may still appear in postgraduate e)ams 9olycystic o0ary syndrome *9C7@+ is a comple) condition of o0arian dysfunction thought to affect #etween 6,134 of women of reproducti0e age. :he aetiology of 9C7@ is not fully understood. Both hyperinsulinaemia and high le0els of luteiniIing hormone are seen in 9C7@ and there appears to #e some o0erlap with the meta#olic syndrome (eatures su#fertility and infertility

menstrual distur#ances$ oligomenorrhea and amenorrhoea hirsuitism& acne *due to hyperandrogenism+ o#esity acanthosis nigricans *due to insulin resistance+ /n0estigations pel0ic ultrasound (@'& L'& prolactin& :@'& and testosterone are useful in0estigations$ raised L'$(@' ratio is a 'classical' feature #ut is no longer thought to #e useful is diagnosis. 9rolactin may #e normal or mildly ele0ated. :estosterone may #e normal or mildly ele0ated , howe0er& if markedly raised consider other causescheck for impaired glucose tolerance Question 13G !ach one of the following is a feature of su#acute thyroiditis& e)cept$ A. Good prognosis B. /ncreased iodine uptake on scan C. 9ainful goitre . 'yperthyroidism !. !le0ated !@" @u#acute * e Quer0ain's+ thyroiditis is associated with decreased iodine uptake on scan @u#acute thyroiditis *also known as e Quer0ain's thyroiditis+ is thought to occur following 0iral infection and typically presents with hyperthyroidism (eatures hyperthyroidism painful goitre raised !@" glo#ally reduced uptake on iodine,1?1 scan ;anagement usually self,limiting , most patients do not re.uire treatment thyroid pain may respond to aspirin or other D@A/ s in more se0ere cases steroids are used& particularly if hypothyroidism de0elops

Das könnte Ihnen auch gefallen